SlideShare a Scribd company logo
1 of 16
Download to read offline
สหพั น ธ์ นิ สิ ตนั กศึ กษาแพทย์ แห่ งประเทศไทย (สพท.)
             National Council of Thai Medical Students (NCTMS)
              วิทยาลัยแพทยศาสตร์กรุงเทพมหานครและวชิรพยาบาล คณะแพทยศาสตร์ มหาวิทยาลัยขอนแก่น               คณะแพทยศาสตร์ จุฬาลงกรณ์มหาวิทยาลัย
              คณะแพทยศาสตร์ มหาวิทยาลัยเชียงใหม่             คณะแพทยศาสตร์ มหาวิทยาลัยธรรมศาสตร์          คณะแพทยศาสตร์ มหาวิทยาลัยนราธิวาสราชนครินทร์
              คณะแพทยศาสตร์ มหาวิทยาลัยนเรศวร                คณะแพทยศาสตร์ มหาวิทยาลัยบูรพา               คณะแพทยศาสตร์พระบรมราชชนก
              วิทยาลัยแพทยศาสตร์พระมงกุฎเกล้า                คณะแพทยศาสตร์ มหาวิทยาลัยมหาสารคาม           วิทยาลัยแพทยศาสตร์ มหาวิทยาลัยรังสิต
              คณะแพทยศาสตร์โรงพยาบาลรามาธิบดี                สํานักวิชาแพทยศาสตร์ มหาวิทยาลัยวลัยลักษณ์   คณะแพทยศาสตร์ มหาวิทยาลัยศรีนครินทรวิโรฒ
              คณะแพทยศาสตร์ศรราชพยาบาล
                              ิิ                             คณะแพทยศาสตร์ มหาวิทยาลัยสงขลานครินทร์       สํานักวิชาแพทยศาสตร์ มหาวิทยาลัยเทคโนโลยีสุรนารี
              วิทยาลัยแพทยศาสตร์และสาธารณสุข มหาวิทยาลัยอุบลราชธานี
            ๓๑๗/๕ ถนนราชวิถี แขวงทุงพญาไท เขตราชเทวี กรุงเทพมหานคร ๑๐๔๐๐ โทรสาร ๐-๒๓๕๔-๗๗๕๐ ต่อ ๑๑๒
                                   ่



Exercise for National License Part II
CLINICAL SCIENCE : March 2010
By NCTMS2010

   แนวข้อสอบชุดนีเป็ นการรวบรวมแนวข้อสอบ
   ทีส่งมาจากโรงเรียนแพทย์ 2 แห่ง ได้แก่

   BM           คณะแพทยศาสตร์วชิรพยาบาล มหาวิทยาลัยกรุงเทพมหานคร
                (วิทยาลัยแพทยศาสตร์กรุงเทพมหานครและวชิรพยาบาล)
   TU           คณะแพทยศาสตร์ มหาวิทยาลัยธรรมศาสตร์

   รวบรวมและเรียบเรียงโดย สโมสรนิสิตคณะแพทยศาสตร์ จุฬาลงกรณ์มหาวิทยาลัย
   ตรวจทานโดย ฝ่ ายวิชาการ สหพันธ์นิสิตนักศึกษาแพทย์แห่ งประเทศไทย รุ่นที 20
   หากมีขอผิดพลาดประการใด ขออภัยมาไว้ ณ ทีนี
         ้
                                                     แนะนําหรือติชมได้ที www.nctms.in.th/webboard


     หมายเหตุ : ข้อสอบฉบับนีเรียบเรียงจาก
                                                                           1. ผู้ป่วยหญิงอายุ 28 ปี GA10 WK HBsAG +ve ตัวเดียวเท่านัน
                                                                              ต้ องแนะนําอย่างไร
                                                                               A. termination
                                                                               B. C/S at term
                                                                               C. avoid breast feeding
                                                                               D. vaccine at birth
                                                                               E. immunoglobulin at birth
                                                                           2. ผู้ป่วยหญิงอายุ 25 ปี G1 GA 20 wk. U/S พบ anencephaly
                                                                              การปฏิบตใดเหมาะสมทีสุด
                                                                                       ั ิ
                                                                               A. ตรวจเลือดแม่ดู AFP
                                                                               B. ตรวจเลือดแม่ดู triple test
                                                                               C. ตรวจ MRI เพือยืนยันการวินจฉัย
                                                                                                             ิ


NLE_step_2_2010_NCTMS ซําซ้อนกัน
         จึ งมีเนือหาส่วนใหญ่                                                                                                        Page 1
D. ตรวจติดตาม U/S อย่างใกล้ ชิด                             8. ผู้ป่วยหญิงอายุ 65 ปี หมดประจําเดือนมา 15 ปี ไม่ได้
      E. แนะนํายุติการตังครรภ์                                       รับประทานฮอร์โมนทดแทน มารพ.ด้ วย AUB 5 วัน จะ
3.   แม่อายุ 40 yr แต่งงานมา 10 yr ไม่ได้ คมกําเนิด GA 22wk
                                              ุ                      Investigate อะไร
     chromosome study 47+xy trisomy 21 จะแนะนําอย่างไร                A. Fractional curragetage
      A. บอกแม่ว่าเป็ นทารกเพศชาย และมีความเสียงในการเกิด             B. Endomethrial biopsy
            ความผิดปกติ                                           9. ผู้ป่วยหญิงอายุ 25 ปี RlQ pain 2 day PTA LMP 7 days ago
      B. บอกแม่ว่าผิดปกติ และวางแผนร่วมกันรักษา                      ,PE tender at RLQ no guarding , PV tender at uterus
      C. U/S หาความผิดปกติ                                           buldging at anexa การวินจฉัยทีถูกต้ องคือ
                                                                                               ิ
4.   หญิงไทยคู่อายุ 24 ปี GA 32 wks มีนาไหลออกจากช่องคลอด
                                            ํ                         A. PID
     3 hr ก่อนมาโรงพยาบาล ไม่เจ็บท้อง speculum exam พบนํา 5           B. appendiditis
     ml, nitrazine test positive ต้ องทําอะไร                     10.ผู้ป่วยหญิงอายุ 30 ปี แต่งงานมาแล้ ว 6 ปี ไม่เคยคุมกําเนิด
      A. oxytocin iv                                                 ปัจจุบนไม่เคยตังครรภ์และมีลูก มีอาการปวดประจําเดือนพอทน
                                                                            ั
      B. terbutaline iv                                              ได้ ไม่ต้องหยุดงาน ตรวจร่างกายพบnormal uterine , mass
      C. C/S                                                         cystic content at Rt. adnexa 3x4 cm. tender , cul-du-sac
      D. dexamethasone im                                            roughly การวินจฉัยทีเหมาะสมคือ
                                                                                     ิ
      E. prostaglandin                                                A. Endrometriosis Cyst
5.   ผู้ป่วยหญิงไทยคู่อายุ 24 ปี G?P? GA 3rd trimesters เจาะถุง       B. Functional Ovarian cyst
     นําครําได้ เป็ นนําใส FHR 80-90 bpm PV ซําพบ pulsatile           C. Ovarian Tumor
     mass การวินจฉัยทีเหมาะสมคือ
                     ิ                                                D. TOA
      A. Vasa previa                                                  E. Appendiceal abscess
      B. Prolapsed cord                                           11.ผู้ป่วยหญิงอายุ 25 ปี คัน ตกขาว ให้ รปมาKOH น่าจะเป็ นพวก
                                                                                                          ู
      C. Placenta previa                                             yeast รักษายังไง?
6.   ผู้ป่วยหญิงอายุ 23 ปี GA 39 Wk พบว่าลูกไม่ดิน 24 Hr. PE:         A. metronidazole oral
     Fundal Height ¾ > Umbilicus, FHR 140/min ข้ อใดเป็ นแนว          B. norfloxacin oral
     ทางการรักษาทีเหมาะทีสุด ในผู้ป่วยรายนี                           C. metronidazole suppo
      A. Reassure and F/U next day                                    D. clotrimazole
      B. Biophysical profile                                      12.ตามมาตรา 276 ประมวลกฎหมายอาญาข้ อใดทีอาจมีความผิด
      C. Oxytocin stress test                                        ฐานข่มขืนกระทําชําเรา(Rape)
      D. Induction of labor                                           A. ผู้ชายใช้ อวัยวะเพศเทียมเพศชายใส่เข้ าช่องปากหญิงอืน
      E. C/S                                                          B. ผู้ชายใช้ อวัยวะเพศเทียมเพศชายใส่เข้ าช่องปากชายอืน
7.   ผู้ป่วยหญิงอายุ 30 ปี ยังไม่มบุตรมาตรวจร่างกาย ไม่มี
                                    ี                                 C. ผู้หญิงใช้ อวัยวะเพศเทียมเพศชายใส่เข้ าช่องปากชายอืน
     เลือดออกผิดปกติ คลํา Uetrus 12 wk sizeU/S Uterus โต 11           D. ผู้ชายใช้ อวัยวะเพศของตนเองถูไถบริเวณทวารหนักของ
     cm,Globular shape,Multiple myoma ก้ อนใหญ่สด 2×2
                                                    ุ                       หญิงอืน
     anterior,2×3 posterior management อย่างไร                        E. ผู้หญิงใช้ อวัยวะเพศชายเทียมสอดใส่เข้ าช่องทวารหนักของ
      A. สังเกตอาการและ F/U 3 mo                                            หญิงอืน
      B. GnRH releasing hormone                                   13.หญิงปัญญาอ่อน ญาติพาแจ้ งความถูกข่มขืนมา3วันก่อนมา
      C. Uterine artery embolization                                 โรงพยาบาล ตรวจร่างกายพบว่ามีอสุจิในช่องคลอด acid
      D. Myomectomy                                                  phosphatase positive มีkissing bite มีแผลถลอกใหม่ทช่อง ี
      E. Hysterectomy


NLE_step_2_2010_NCTMS                                                                                            Page 2
คลอด และ รอยฉีกขาดใหม่ทางด้ านหลังของช่องคลอด ถามว่า                กลับมาทํางานได้ ตามปกติ ตํารวจตังข้ อหาและศาลสังฟ้ องฝ่ าย
   อะไรทีทําให้ แพทย์ลงความเห็นว่า ถูกกระทําชําเรามา 3วัน              ชายว่าทําร้ ายร่างกายจนสาหัส โดยใช้ เหตุผลข้ อใดตามประมวล
    A. พบอสุจในช่องคลอด
                  ิ                                                    กฎหมายอาญา มาตรา 276
    B. Acid phosphatase positive                                        A. ตามองไม่เห็น
    C. พบkissing bite                                                   B. แผลบริเวณใบหน้ า
    D. มีรอยถลอกทีช่องคลอด                                              C. แผลฟกชําทีท้ องน้ อย
    E. มีรอยฉีกขาดทีทางด้ านหลังของช่องคลอด                             D. เลือดออกทางช่องคลอด
14.ผู้ป่วยหญิงไทยอายุ 15 ปี เคยมีระดูมาแล้ ว มีอาการขาดระดูมา           E. รอยชําทีเบ้ าตา
   2 สัปดาห์ ประวัตถูกข่มขืนมา 5 ชัวโมง ตรวจร่างกายพบรอย
                     ิ                                                  F. บาดแผลขอบเรียบ 10 ซม.
   kiss mark ทีคอ มีรอยฟกชําทีบริเวณท้ องน้ อย ตรวจภายในพบ              G. ผล pathology ได้ Chrorionic villi
                                                                    18.ผู้ป่วยชายไทยอายุ 80 ปี U/D DM, HT, CKD, IHD อาศัยอยู่
   รอยฟกชํารอบอวัยวะเพศ และพบมูกสีขาวใสในช่องคลอด ลง
                                                                       กับภรรยาอายุ 75 ปี เพียงลําพัง ปฏิเสธการย้ ายไปอยู่กบ  ั
   ความเห็นว่าน่าจะถูกกระทําชําเราภายในเวลา 3 วัน จะ                   ลูกหลานเพราะผู้ป่วยอยากทําอะไรด้ วยตนเอง ถึงแม้ ว่าในขณะนี
   Management อย่างไร                                                  จะให้ ภรรยาช่วยจัดยาให้ ช่วยพยุงเดินช่วง 2-3 เดือนมานีผู้ป่วย
    A. ส่ง acid phosphatase                                            มีอาการเบือหน่ายไม่อยากทําอะไร นําหนักลด เบืออาหาร บ่นว่า
    B. Urine pregnancy test                                            อยากตายอยู่บ่อย ๆ มีอยู่วนหนึงผู้ป่วยเกิดหัวใจวายกะทันหัน
                                                                                                   ั
    C. ให้ Norfloxacin                                                 ญาตินาส่งรพ. หลังจากกู้ชีวต ฟื นฟูสภาพ ล้ างไตและกลับบ้ าน
                                                                               ํ                     ิ
    D. ให้ Levonorgestrel                                              ได้ แต่ยงต้ องมาล้ างไต 3/สัปดาห์ ครังนีผู้ป่วยขอให้ แพทย์หยุด
                                                                                 ั
    E. แนะนําให้ กลับมารักษาหากยังขาดระดูอยู่                          ล้ างไต และขอให้ ตนจากไปอย่างสงบ แพทย์ควรทําอย่างไรกับ
15.ศพผู้ชายนอนเสียชีวตอยู่ข้างบาทวิถี ตรวจพบแผลขอบไม่เรียบที
                       ิ                                               ผู้ป่วยรายนี
   ท้ ายทอย, กะโหลกศีรษะด้ านหลังแตกยาวประมาณ 5 cm. มี                  A. ทําตามทีผู้ป่วยต้ องการ และให้ เซ็นปฏิเสธการรักษา
   brain contusion ด้ านหน้ ามากกว่าด้ านหลัง ถามสาเหตุการ              B. แจ้ งญาติเกียวกับความต้ องการของผู้ป่วยและเซ็นรับรอง
   เสียชีวต
          ิ                                                                   ความต้ องการ
    A. ถูกตีด้วยของแข็งทีท้ ายทอย                                       C. แจ้ งลูกให้ รับรู้ความต้ องการของผู้ป่วยและตัดสินใจ
    B. ล้ มท้ ายทอยกระแทกขอบบาทวิถี                                     D. แจ้ งภรรยาเกียวกับความต้ องการและตัดสินใจ
    C. ถูกตีด้วยของแข็งทีหน้ าผาก                                       E. พูดคุยกับผู้ป่วยและรักษาอาการซึม
    D. ล้ มหน้ าผากกระแทกขอบบาทวิถี                                 19.ผู้ป่วยเด็กชายอายุ 1 ปี มีไข้ ซีด อย่างรวดเร็ว PE: mild pale,
    E. ถูกตีทท้ายทอยและหน้ าผาก
                ี                                                      moderate jaundice, hepatosplenomegaly Lab: Hct 21%
16.ศาลยกฟ้ องโจทก์คดี จําเลยโดนยิงด้ วยปื นลูกโดด ถามว่า               WBC 13800 Plt 245000 , PBS: hypochromic microcytic
   บาดแผลเป็ นอย่างไร                                                  RBC, marked anisocytosis + poikilocytosis จะส่งตรวจอะไร
    A. กลม มีรอยไหม้                                                   เพือช่วยในการวินจฉัย
                                                                                          ิ
    B. รี ถลอกมีรอยไหม้                                                 A. G6PD screening
    C. Star shape                                                       B. Hb typing
17.การตรวจร่างกายหญิงรายหนึงทีถูกทําร้ ายร่างกายโดยฝ่ ายชาย             C. Coomb's test
   ตรวจร่างกายพบว่าตาขวามองไม่เห็นเนืองจากลืมตาไม่ขน มีรอย ึ            D. serum ferritin
   ฟกชําทีเบ้ าตา มีบาดแผลขอบเรียบบริเวณใบหน้ าข้ างขวาขนาด             E. OF test
   10 cm. มีแผลฟกชําบริเวณท้ องน้ อย ตรวจพบเลือดออกทาง              20.ผู้ป่วยเด็กชายอายุ 7 ปี มีไข้ ไอ เจ็บคอ นํามูกเล็กน้ อยมา 2 วัน
   ช่องคลอดและมีก้อนเลือดหลุดออกมาด้ วย ผลตรวจทางพยาธิ                 ซือยากินเอง วันนีมีไข้ สง ซีด ปัสสาวะสีดา ตรวจร่างกายพบ
                                                                                                ู               ํ
   วิทยาพบเป็ น chorionic villi ผู้ป่วยรักษาตัวอยู่ 15 วัน สามารถ      moderate pale, no jaundice, no hepatosplenomegaly, pharynx


NLE_step_2_2010_NCTMS                                                                                                 Page 3
and tonsils are injected ส่ง investigation ใดเหมาะสมทีสุดใน       25.ผู้ป่วยเด็กชายอายุ 2 ปี กินเก่งมากขึนมา 2 เดือน V/S BP
   การวินจฉัย
           ิ                                                            140/100, HR 100, RR 24, BW 25 kg, Ht 90 cm GA -
    A. UA                                                               Generalized obesity, Hirsutism Heart&Lung – WNL Abd -
    B. reticulocyte count                                               purplish striae จงให้ การวินจฉัยผู้ป่วยรายนี
                                                                                                    ิ
    C. complete blood count                                              A. Exogenous steroid
    D. G6PD level                                                        B. Prader-Willi's Syndrome
    E. peripheral blood smear                                            C. Neuroblastoma
21.ผู้ป่วยอายุ 5 ปี มจุดเลือดออกตามตัว ไม่มไข้ PE: Generalized
                     ี                        ี                          D. Pheochromocytoma
   multiple supreficial ecchymosis, otherwhise is                        E. Cushing Syndrome
   normal ,CBC: Hct 34-37%,WBC 6000,N 60-65%,L                       26.ผู้ป่วยเด็กชายอายุ 5 ปี มีอาการไข้ สงทานยาลดไข้ แล้ วไม่ลดลง
                                                                                                              ู
   20-27%,E 12-15%,plt 200000,Bleeding time >15 min                     มา 5 วัน ไอ ปวดหัว เพลีย ปวดท้ อง มีอาการคลืนไส้ อาเจียน
   ให้ management                                                       3-4 ครัง มีผนตามร่างกาย ตรวจร่างกาย วันนีไข้ ลง BT 37
                                                                                          ื
    A. ส่ง ANA                                                          BP 90/70 mmHg PR 120/min RR 32/min BW 20, look
    B. stool occult blood                                               weak, flush face, liver 4 cm BRCM petechiae both arms and
    C. Pednisolone                                                      legs จงให้ การรักษาทีเหมาะสมในผู้ป่วยรายนี
    D. Platelet transfusion                                              A. 5% DN/3 rate 60 cc/hr
    E. Cryoprecipitate                                                   B. 5% DN/2 rate 100 cc/hr
22.ผู้ป่วยเด็กชายอายุ 16 ปี จําเลือดเป็ นๆหายๆ ครังนีมาด้ วยมีเข่า       C. 5% DN/2 rate 200 cc/hr
   บวม 1วัน PE: warm+swelling right knee Lab: CBC,Plt ไม่ตา      ํ       D. 5% DNSS rate 100 cc/hr
   PT ปกติ PTT 68 manage เบืองต้ น???                                    E. 5% DNSS rate 200 cc/hr
    A. DDAVP                                                         27.ผู้ป่วยเด็กชายอายุ 4 ปี จมนํา ไม่ร้ สติ PE : mild cyanosis BT
                                                                                                            ู
    B. Cryo                                                             36.5-37.5 BP 90/60 HR 140 RR 50 fine crep+wheeze
    C. FFP                                                              both lung O2sat (O2 10 LPM ) = 90% จง manage
    D. Cryo-removed plasma                                               A. NB beta2 agonist
    E. Factor VII                                                        B. NB adrenaline
23.ผู้ป่วยเด็กชายอายุ 8 ปี มีผนขึนตัวลามไปแขนขา หลังทาน
                               ื                                         C. IV dopamine
   อาหาร BP 80/50ปวดท้ องอาเจียน lung: no wheezing                       D. O2 mask c bag
   Mx???                                                                 E. Intubation c ventilator support
    A. Adrenaline IM ***                                                 F. NSS
    B. CPM IV                                                        28.ผู้ป่วยเด็กชายอายุ 5 ขวบ ถูกนําส่งมาด้ วยทีมีอาการจมนําในสระ
24.ท่านเป็ นแพทย์ประจํารพ. ผู้ปกครองมาขอปรึกษาเมือ 3                    ว่ายนํา ไม่ร้ สกตัว ตรวจร่างกายพบ central and peripheral
                                                                                      ู ึ
   สัปดาห์หลังจากรับทราบผลการตรวจเลือดของลูกตอนแรก                      cyanosis ให้ initial management
   คลอด 3 วัน ว่ามี TSH ผิดปกติ...ท่านจะทําอย่างไร                       A. CPR
    A. เจาะ thyroid antibody ของแม่                                      B. Respiration support
    B. เจาะ T4, TSH ลูก                                              29.ผู้ป่วยเด็กชาย preterm BW = 2200 gm มารดาคลอด C/S
    C. เจาะ thyroglobulin ของลูก                                        due to abruption placenta apgar score 5, 8 หลังคลอด 12 ชม
    D. ทํา thyroid ultrasonography                                      เหลือง ทํา exchange สาหตุสาคัญของการทีต้ องตรวจการได้ ยิน
                                                                                                       ํ
    E. ให้ thyroxine                                                    ของเด็กคนนี คือ
                                                                         A. BW = 2200


NLE_step_2_2010_NCTMS                                                                                                 Page 4
B. abruptio placenta                                              D. Chest compression
    C. apgar score 5,8                                                E. Adrenaline iv
    D. preterm                                                    36.เด็กเหม่อ อาการ Absance รักษาอย่างไร
    E. เหลือง                                                                Ans. Na valproate
30.เด็กครบกําเนิด หนัก3400กรัม เหลืองที10ชัวโมง MB                37.เด็กแรกคลอด ไม่ได้ LMP 12 Hr หลังคลอด มีหายใจหอบ
   10 hct 38 เด็กเลือดกรุ๊ป เอ direct coomb weekly+ve แม่            เหนือย BW 3200 RR 60 bpm ผิวหนังเหียวย่น เล็บยาว มี
   เลือดกรุป โอ indirect coomb –ve ทําอย่างไร
            ๊                                                        meconium stain fluid Hct70% DTX 50 ถามว่าเด็กคนนีเป็ น
    A. Hydration                                                     อะไร
    B. PRC transfusion                                                A. Pneumonia
    C. exchang transfusion                                            B. Polycythemia
    D. intensive phototherapy                                         C. Hypoglycemia
    E. avoid breast feeding                                           D. RDS
31.NB 6 hr ซึมลง MAP 38 mmHg ทําไร                                    E. MAS
    A. Normal saline iv load                                      38.ผู้ป่วยเด็กชายอายุ 1 ขวบ ไอเสียงก้ อง รักษา?
    B. Dobutamine                                                            Ans.Epinephrine NB
32.NB หนัก 4200 gm ที 2 ชม.มีอาการกระตุก แขน ขา เจาะ BS           39.ผู้ป่วยเด็กชายอายุ 10 เดือน มีไข้ ประมาณ 38 มีหายใจขอบ
   ได้ 37 ให้ 10%glucose จนเพิมเป็ น 25% glucose Plasma              เหนือย หายใจเร็ว ฟัง lung ได้ wheezing มาพบแพทย์ ได้ พ่น
   glucose 50 mg/dl น่าจะเกิดจากสาเหตุใดทีสุด                        ยาขยายหลอดลมไป สองครัง อาการไม่ดขน นึกถึงอะไร
                                                                                                             ีึ
    A. Hypopituitary                                                  A. Asthma
    B. hyperinsulinemia                                               B. Pneumonia
    C. Hypercortisol                                                  C. Croup
    D. Adrenal insufficiency                                          D. bronchiolitis
    E. Alocohol withdrawal syndrome                               40.ผู้ป่วยเด็กชายอายุ4ปี พ่อเป็ นTB on ยา อยู่ 1 สัปดาห์กอน ลูก
                                                                                                                           ่
33.Case TOF เขียว เหนือย central cyanosis Culture→Entero             ไม่มอาการอะไรเลย ตรวจร่างกายปกติ ตรวจPPD negative ถาม
                                                                           ี
   spp.ให้ ATB?                                                      management?
          Ans.Vancomycin                                              A. ไม่ทาไรเลย
                                                                                ํ
34.ผู้ป่วยเด็กชายอายุ 1 ปี มีอาการชัก เขียว หมดสติ ตรวจร่างกาย        B. INH prophylaxis 3 mo
   พบ flacid, cyanosis, no heart sound ควรทําสิงใดเป็ นอันดับ         C. INH prophylaxis 9 mo
   แรก                                                                D. INH + rifampin 3 mo
    A. Defibrillation                                                 E. INH + rifampin 9 mo
    B. chest compression                                          41.ผู้ป่วยชายอายุ 18 yr หมดสติ หลังงานรับน้ อง ได้ กลินเหล้ า PR
    C. ambu bag with mask 100%O2                                     60 BP100/60
    D. IV adrenaline                                                  A. Ketone level
    E. intubation                                                     B. Electrolyte
35.เด็ก 1 ปี ชักเกร็ง ส่งER ไม่ร้ สกตัว Cyanosis no heart sound
                                  ูึ                                  C. CT brain
   ถาม First management ?                                             D. 50% glucose IV
    A. O2 mask 100%                                                   E. Atropine IV
    B. ET tube                                                    42.ผู้ป่วยชายอายุ 77 ปี หมดสติ คลํา carotid pulse ไม่ได้ ทํา chest
    C. Defibrillation                                                compression ไป ekg : asystole ทําอะไรต่อ


NLE_step_2_2010_NCTMS                                                                                               Page 5
A. Adrenaline***                                            47.ผู้ป่วยหญิงอายุ 65 yr U/D HTN ,glaucoma ซือยากินเอง ญาติ
    B. Amiodarone                                                  พบหมดสติ ไม่ร้ สกตัว PE: ซึม, BP 140/90 ,disorentated to
                                                                                     ุึ
    C. Defrib 200 J                                                time-place-person, Lab : hypoNa ,hypoK, hypoCl, CO2 35
    D. Cardioverse 100 J                                           BUN 40 Cr 1.6 ถามว่าอาการดังกล่าวเกิดจากยาตัวใด
43.ผู้ป่วยหญิงอายุ 18 ปี มีเลือดออกตามไรฟัน ประจําเดือนมานาน        A. acetazolamide
   กว่าปกติ มีจุดเลือดออกทีขา ไม่ซด ตับม้ ามไม่โต CBC Hb12.2
                                    ี                               B. HCTZ
   WBC5000 Plt50000 PTT40 PT30 ถามว่าเป็ นอะไร                      C. spinorolactone
    A. MDS                                                      48.ผู้ช่วยพยาบาลประจําห้ องฉุกเฉินโดนเข็มตําจากการเจาะเลือด
    B. DIC                                                         ผู้ป่วยเมาสุรา มีเลือดออกเล็กน้ อย ได้ ทาแผลและใส่ betadine
                                                                                                           ํ
    C. TTP                                                         แล้ ว การปฏิบติตัวทีเหมาะสม สําหรับผู้ช่วยพยาบาลรายนีคือข้ อ
                                                                                  ั
    D. ITP                                                         ใด
44.ผู้ป่วยชายอายุ 18 ปี ดืมนํามาก ปัสสาวะมาก 2เดือน ปวดหัว          A. ขอผู้ป่วยตรวจเลือดหา Anti-HIV
   ตามัว 1เดือน MRI brain : Suprasellar mass E’lyte : Na 145        B. เริมกินยา Antiviral ได้ ทนที
                                                                                                 ั
   K 3.8 Cl 110 CO2 21 fluid input/output =                         C. รอตรวจเลือดซําอีก 6 สัปดาห์
   5100:5800manage                                                  D. ส่งเลือดผู้ป่วยตรวจหา Anti-HIV
    A. Restrict fluid                                               E. แนะนําว่าโอกาสติดเชือน้ อยมาก
    B. Restrict Na                                              49.ชาย 75 ปี เก็บเหล็กขาย มีอาการนอนไม่หลับ กระวนกระวาย
    C. Hydrocortisone                                              กระส่าย กระสับ ปวดท้องรุนแรงเป็ นพักๆ 3เดือน PE: marked
    D. Hctzz                                                       pale Ix??
    E. DDAVP                                                        A. LFT
45.ผู้ป่วยชายอายุ 25 ปี ไอเรือรัง AFB 3+ ,CXR : Cavity with         B. Blood lead level***
   surrounding granuloma Dx Pulmonary TB ได้ IRZE 2 เดือน           C. EKG
   F/U ได้ AFB 1+ ทําไรต่อ                                          D. CT Abdomen
    A. ลดเหลือ IR                                               50.ผู้ป่วยชายอายุ 30 ปี เหนือย อ่อนเพลีย BP 160/110 PR 70
    B. ลดเหลือ IRE                                                 อืนๆปกติ Na 154 K 2.2 HCO3 32 การวินจฉัยทีเหมาะสมคือ
                                                                                                               ิ
    C. ให้ IRZE ต่อ 1 เดือน                                         A. Essential HT
    D. Add streptomycin                                             B. Hyperthyroid
    E. Add ofloxacin                                                C. Pheochromocytoma
46.ผู้ป่วยหญิงอายุ 25 ปี ไข้ ไอ เหนือย 5 วัน ไอเสมหะเหลือง ไป       D. 1st hyperaldosterone
   ซือ Amoxycillin500 1×3 กินเอง 3 วัน ไม่ดีขน PE:
                                                ึ                   E. Cushing syndrome
   BT36.5°C,PR 102,RR 26,BP135/70Lung:Dullness on               51.ผู้ป่วยหญิงอายุ 68 yr U/D HT กินยาไม่สมําเสมอ มาด้ วยหอบ
   percussion right lower lung and decrease breath sound and       เหนือย PE: BP200/110 PR 100 neck vein engorge S3
   vocal resonance right lower lung รักษาอย่างไร                   gallop fine crep both lung Ext: pitting edema lower leg Eye:
    A. เพิม Amoxycillin                                            exudates c wire ring Mx??
    B. เพิม Clarithromycin                                          A. Digoxin
    C. เปลียนเป็ น Levofloxacin                                     B. Propanolol
    D. Thoracentesis                                                C. Nifedipine
    E. ICD                                                          D. Nitroglycerine



NLE_step_2_2010_NCTMS                                                                                           Page 6
52.DM ได้ MFM 1500,Glipizide 15 BMI เยอะ FBS 180                          Ans.Indommmethacin
   HbA1C 8.1                                                   62. HIV เป็ น Meningitis CSF glucose ลด Protein เพิม CD4>250
    A. เพิม Glipizide                                              Indian ink neg การวินจฉัยทีเหมาะสมคือ
                                                                                          ิ
    B. MFM 2000                                                           Ans.TB meningitis
    C. add acarbose                                            63. ผู้ชายปวดหัว ปวดเบ้ าตาจนต้ องตืนกลางดึก มีอาการนาน 2
    D. ให้ RI                                                      เดือน แล้ วหายไป ตอนนีกลับมาเป็ นใหม่มีปวดเบ้ าตาซ้ ายบ้ าง
53.ผู้ป่วยชายอายุ 70 ปี CC: ผล Lab ผิดปกติ ทานวิตามินเสริม         หรือเบ้าตาขวาอย่างเดียว การวินจฉัยทีเหมาะสมคือ
                                                                                                    ิ
   เป็ นประจํา 10 ปี ก่อนเคยปัสสาวะปนทราย PTH 7, Ca2+11,            A. Cluster headache
   PO4 2- 3.5, Cr 1.1, การวินจฉัยทีเหมาะสมคือ
                                  ิ                                 B. Tension headache
    A. HyperCa2+ of Malignancgy                                     C. Migraine
    B. CA with bone Met.                                       64.SLE เป็ น Pregressive GN on cyclophosphamide แล้ วมีไข้ ตุ่ม
    C. 1OHyperparathyroid                                          แดงต่อมามีสดาตรงกลางคล้ าย Eschar รักษา?
                                                                                ี ํ
    D. 2OHyperparathyroid                                                 Ans.Doxycyclin
    E. 3OHyperparathyroid                                      65. DM HT DLP กินยาบํารุงหลายอย่าง PTH สูง Ca สูง
54.EKG bradycardia Mx                                               A. Vit D intox
          Ans:Atropine                                              B. 1° hyperparathyroid
55.กินหน่อไม้ ดอง + เหล้ า เกิด Respiratory…                        C. 2° hyperparathyroid
          Ans : Botulinum toxin                                     D. 3° hyperparathyroid
56.EKG Asystole Mx                                             66. รูปงู ผล Lab ปกติมี Local wound swelling,Hemorrhagic bleb
          Ans:Adrenaline                                           ทําไร
57.Hypo K, Hypo Na                                                  A. Antivenum
          Ans : 1o Hyperaldosteronism                               B. TT
58.ผู้ป่วยหญิงอายุ 80 ปี มาด้ วยอ่อนแรงซีกซ้ าย หมดสติ 5 day        C. Broad spectrum ATB
   PTA ขณะนังดูโทรทัศน์ PE: stupor HR 60 totally irregular     67.ผู้ป่วยชายอายุ 70 ปี ไม่มโรคประจําตัว lipid profile TG 500
                                                                                             ี
   RR 12 Lt.hemiparesis pupil 2 mm Lt. 4 mm Rt. Upgoing            Choles 210 HDL 40 Lab อะไรผิดปกติด้วย
   Lt.plantar reflex ท่านเป็ นแพทย์ที ER จง Manage เบืองต้ น        A. Uric acid
   อย่างไร                                                          B. FBS
    A. IV furosemide                                                C. Cortisol
    B. IV nitroglycerine                                            D. Thyroid function test
    C. IV heparin drip                                         68. ผู้ป่วยหญิงอายุ 16 ปี ล่องเรือไปเทียวเกาะ ขณะเรือออกกลาง
    D. Et tube                                                     ทะเลมีคลืนไส้ อาเจียน ใช้ ยาอะไร
    E. IV Dexamethasone                                             A. Paracetamol
59.ผู้หญิงมีรอยนําตาลนูนมา 5 วัน ตรงกลางมีผมร่วงในแผลเห็น           B. Dimenhydrinate
   เส้ น เลือดชัดเจน (Telangiectasia)                               C. Propanolol
          Ans.Discoid LE                                            D. tramadol
60.ผู้ป่วยหญิงอายุ 65 ปี DM HT on ยา HCTZ ACEI MFM แล้ ว            E. Cinnarizine
   กิน Allopurinol Para แล้ วมีผน MP rash เกิดจาก?
                                ื                              69.ผู้ป่วยหญิงอายุ 75 ปี U/D HT on ยา HCTZ (25) 1x1
          Ans.Allopurinol                                          ควบคุมความดันได้ ปกติ ตอนเช้ าลุกจากทีนอนไม่ขน ญาตินาส่ง
                                                                                                                    ึ       ํ
61.อาการ Gout เป็ นครังแรก รักษา?                                  รพ. PE: Alert, BP 130/80 mmHg, PR 72, RR 18, BT 37


NLE_step_2_2010_NCTMS                                                                                          Page 7
Proximal m. weakness, normal sensation and reflex อาการ               C. ASA + CCB
    ของผู้ป่วยน่าจะเกิดจากอะไร                                            D. ASA + BZD
     A. HypoNa+                                                           E. beta-block + heparin
     B. HypoK+                                                       74. ผู้ป่วยหญิง ซีด อ่อนเพลีย CBC เป็ น iron def ได้ รับการรักษา
     C. HypoMg2+                                                         ด้ วยยาเม็ดสีนาตาลดํา ถามว่าหลังได้ รับยาจะติดตามการรักษา
                                                                                         ํ
     D. HyperCa2+                                                        ด้ วยวิธใดได้ เร็วทีสุด
                                                                                 ี
     E. Rhabdomyolysis                                                    A. Hb
70. ผู้ป่วยชายอายุ 65 ปี เป็ นมะเร็งปอดระยะที 1 ได้ รับการผ่าตัด          B. MCV
    ไปแล้ ว หลังผ่าไม่ได้ รับเคมีบาบัดหรือรังสีรักษา
                                    ํ                                     C. Recticulocyte count
    ต่อมามีอาการซีกขวาอ่อนแรง gr.4 ร่วมกับมีอาการชักเกร็งทัว              D. Serum ferritin
    ตัว ก่อนมา 30 นาที ชักเกร็งกระตุกจากแขนขา แล้ วต่อมาชักทัว            E. Serum haptoglobin
    ตัว หลังชักรู้สกตัวดี PE : good conciousness , Lung - clear ,
                   ึ                                                 75.ผู้ป่วยชายไทยมีไข้ และเจ็บคอมา 1 wk ซือยามากินเองอาการไข้
    scar at Lt. chest wall , tracheotomy scar จะให้ ยาอะไร               ดีขนแต่มอาการอ่อนเพลียมากขีนมา 3 d PE: v/s ปกติ , pale
                                                                               ึ   ี
     A. Clonazepam                                                       conjunctiva, icteric sclera , spleen 2 FB BLCM LAB: Hct
     B. Phenobarbital                                                    25% MCV 65 plt 400000 WBC 9600 N 60% L 40%
     C. Phenytoin                                                        ถามว่า diagnosis อะไร
     D. Carbamazepine                                                     A. Hb H disrase
71. ผู้ป่วยชายอายุ 75 ปี no underlying. ไม่ได้ รับประทานยาใด              B. G6PD
    รับประทานอาหารได้ ไม่มนาหนักลด ท้องผูกเป็ นประจํา ตรวจ
                                ี ํ                                       C. HS
    ร่างกายพบ V/S BP 140/80 PR 84 RR 18 (mild)pale,                       D. AIHA
    total dental loss, glossitis, koilonychiae จงให้ การวินิจฉัยที        E. PNH
    เป็ นไปได้ มากทีสุด                                              76.ผู้ป่วยหญิงอายุ 30 ปี มาด้ วยอ่อนเพลีย ซีด 3 สัปดาห์ ตรวจ
     A. hypothyroid                                                      ร่างกายพบ moderate pale, mild icteric sclera, other within
     B. normal aging change                                              normal CBC : Hct 25%, WBC 5000 cell/mm3 N68 L32,
     C. iron deficiency anemia                                           plt. 200000 cell/mm3, RBC morphology : normochromic,
     D. anemia of chronic disease                                        anisocytosis 2+, microspherocyte 2+, polychromasia 1+ การ
     E. autoimmune hemolytic anemia                                      ตรวจใดช่วยยืนยันการวินจฉัยิ
72. ให้ ผล csf : open pressure 30 cmh20 protein 550 sugar 20              A. Hb typing
    ผู้ป่วยมาด้ วยอาการไข้ ปวดศีรษะ การวินจฉัยทีเหมาะสมคือ
                                                ิ                         B. Coomb’s test
     A. toxoplasmosis meningitis                                          C. G6PD
     B. CMV meningitis                                                    D. Osmotic fragility
     C. tubeculous meningitis                                        77.ผู้ป่วยหญิงอายุ 70 ปี มีอาการ อ่อนเพลีย เหนือย มา 4-5
73. ผู้ป่วยชายอายุ 37 yr เจ็บกลางอกร้ าวไปไหล่ซ้าย เหงือแตก ใจ           เดือน ตรวจร่างกายพบ glossitis, moderate pale, CBC : Hct
    สัน มีประวัติเป็ นโรคหืด ใช้ ยาขยายหลอดลมนานๆครัง ไม่มี              22%, WBC 3400, N 65%, L 30%, Plt 80000, ให้ รป          ู
    บุคคลในครอบครัวเป็ นโรคหัวใจ PE: BP 170/85 PR 110                    blood smear มา เหมือนมี hypersegmented neutrophil การ
                                                                         วินจฉัยทีเหมาะสมคือ
                                                                             ิ
    EKG ST elevation V2-6 การดูแลขันต้ นในผป รายนี?
                                                                          A. Aplastic anemia
     A. ASA + beta-block                                                  B. Megaloblastic anemia
     B. ASA + nitrate                                                     C. Iron deficiency anemia


NLE_step_2_2010_NCTMS                                                                                                 Page 8
D. Myelodysplasia                                                   E. PE
78.ผู้ป่วยชาย อ่อนแรงมากขึนมา 3 วัน ตรวจร่างกายพบ total             82.ให้ รป blood smear มา ร่วมกับให้ อาการของ malaria มา blood
                                                                             ู
   opthalmoplegia, bilateral facial palsy, paradoxical breathing,      smear มีลกษณะของ double chromatin and multiple ring บอก
                                                                                            ั
   generalized muscle weakness grade II/V, areflexia, loss of          ถึง P.f. ถามว่าจะรักษาด้ วยยาอะไร
   proprioception of fingers and toes. What is the most likely                   ตอบ artesunate + mefloquine
   diagnosis                                                        83.หญิงอายุ 55 ปี มี U/D เป็ นโรคไตเรือรัง ฟอกไต 2 ครัง/
    A. Multiple sclerosis                                              สัปดาห์ มาด้ วยหอบเหนือย Electrolyte : Na 138, K 6.8,
    B. Myasthenia Gravis                                               BUN 60, Cr 7...รักษาอย่างไร
    C. Guellain Barre Syndrome                                          A. NaHCO3
    D. Acute myelopathy                                                 B. Insulin + นําตาล
    E. Botulism                                                         C. Dialysis
                                                                        D. Beradual
79.ผู้ป่วยชายไทยอายุ28ปี นําหนัก 80กิโล ประสบอุบติเหตุทาง
                                                      ั
                                                                        E. Kayexalate
   รถจักรยานยนต์ ได้ รับอุบตเหตุทสมอง นอนพักรักษาตัวที
                               ั ิ  ี
                                                                    84.ผู้ป่วยชายอายุ 75 ปี มีอาการมือสันเวลาถือแก้ วนํา เป็ นมากขึน
   โรงพยาบาลมาเป็ นเวลา 7วัน ได้ รับอาหารผ่านทางNG
                                                                       เวลาโมโหหรือดืมกาแฟ PE: postural tremor , normal muscle
   tube มาตลอด โดยได้ พลังงาน 1800 kcal/day และ protein
                                                                       tone, no dysmentia, normal gait นึกถึงภาวะใดมากทีสุด
   90 g/day ต่อมาพบว่าalbuminลดลงจาก 4.2เหลือ 2.4 ถาม
                                                                        A. normal change
   ว่าควรปรับเปลียนอาหารอย่างไร
                                                                        B. Parkinson
    A. เพิมพลังงานเป็ น 2000 เพิมโปรตีนเป็ น 120
                                                                        C. cerebellar dystrophy
    B. เพิมพลังงานเป็ น 2400 โปรตีนเป็ น 120
                                                                        D. autoimmune dysfunction
    C. เพิมพลังงานเป็ น 2800 โปรตีนเป็ น 140
                                                                    85.ผู้ป่วยหญิงอายุ 23 ปี อาชีพครูโรงเรียนเด็กอนุบาล เคยป่ วยเป็ น
    D. เพิมไข่ขาว 2 ฟองต่อมือ
                                                                       Rheumatic fever ตอนอายุ 15 ปั ปัจจุบนทํางานทัวไปได้ ปกติ
                                                                                                               ั
    E. เพิมไข่ขาวและไข่แดงมือละฟอง
                                                                       ไม่มอาการหอบเหนือย ตรวจร่างกายพบ pansystolic murmur
                                                                               ี
80.ผู้ป่วยหญิงอายุ 40 ปี จะไปเทียวภูเขาหิมาลัย มาพบแพทย์เพือ
                                                                       grade II/VI at apex ได้ รับการรักษาด้ วย Penicillin V มาตลอด
   ขอคําปรึกษาและยา เพือลดอาการคลืนไส้ ขณะไปเทียว ข้ อใด
                                                                       ควรให้ การรักษาผู้ป่วยรายนีต่อไปอย่างไร
   ควรปฏิบติมากทีสุด
            ั
                                                                        A. หยุดยา penicillin
    A. omeprazole
                                                                        B. ให้ กนยาเฉพาะตอนทําฟั น
                                                                                      ิ
    B. Altitude acclimation
                                                                        C. ให้ กนยาไปจนกระทังอายุ 30 ปี จึงหยุด
                                                                                        ิ
    C. Chloroquine
                                                                        D. ให้ กนยาไปจนกระทังอายุ 50 ปี จงหยุด
                                                                                    ิ                        ึ
    D. Hydrochlorothiazide
                                                                        E. ให้ กนยาไปตลอดชีวต
                                                                                          ิ       ิ
    E. Metformin
                                                                    86.หญิง มาหาหมอ ปรึกษาเรืองไปเทียวหิมาลัย แล้ วไม่อยาก
81.ผู้ป่วยหญิงอายุ 20 ปี มีไข้ มาสามวัน ต่อมามีเจ็บแน่นหน้ าอก
                                                                       คลืนไส้ อาเจียน
   เหนือยง่ายขึน นอนราบไม่ได้ ตรวจร่างกาย V/S : BT 39, BP
                                                                        A. Melatonin
   90/70,PR110 ,RR22 neck vein engorge, decrease heart
                                                                        B. Thiazide
   sound w/ scattering, lung clear ,other WNL
                                                                        C. Omeprazole
    A. CHF
                                                                        D. Chroroquine
    B. Severs MS
                                                                        E. Attitude acclimator
    C. Pericarditis
                                                                    87.HBsAg + HBcIg M + Anti HBe - LFT normal Mx???
    D. Myocarditis
                                                                        A. F/U 6 mo


NLE_step_2_2010_NCTMS                                                                                                 Page 9
B. Start lamivudine                                             92.ผู้ป่วยชายอายุ 70 ปี เป็ น pneumonia มา 5 วัน มี U/D COPD
    C. Viral load                                                      ให้ O2 5 LPM Cef-3+azithromicin อยู่ๆก็เหนือยมาก
    D. Reassure                                                        ขึน และซึมลง ควรทําอย่างไร
88.ผู้ป่วยชายอายุ 45 ปี กินเหล้ า มีไข้ และเจ็บอก 1 wk ตรวจ             A. intubation
   ร่างกายพบ pleural effusion จึงเจาะปอดได้ นาสีเหลืองขาวขุ่น
                                                ํ                   93.ผู้ป่วยชายอายุ 60 ปี มาอยู่ ICU Dx pneumococcal
   ย้ อมแกรมได้ Gram negative bacilli, Gram positive cocci in          pneumonia มา 10 วัน อาการดีขน เมือวานสําลักอาหาร วันนี
                                                                                                          ึ
   chain , positive bacilli จะให้ ยาตัวใด                              มีไข้ สง มีเสมหะและไอ BT 39.5 ,BP 90/60, PR 100, RR
                                                                              ู
    A. penicillin                                                      30 Imaging : crepitaion at Lt. lower lung, Sputum G/S :
    B. ceftriaxone                                                     Gram neg. bipolar stain ให้ ATB อะไร?
    C. cotrimoxazole                                                    A. Amikacin
    D. clarithomycin                                                    B. Amoxy+Clav.
    E. amoxyclav                                                        C. Ciprofloxacin
    F. Levofloxacin                                                     D. Imipenem
89.ผู้ป่วยชายอายุ 32 ปี หอบเหนือยมา 3 วัน เป็ นหลังไข้ หวัด เป็ น       E. Ceftriaxone
   โรคหืดมา 20 ปี ปัจจุบันได้ inhale budesonide PE: tachypnea,      94.ผู้ป่วยชายอายุ 58 ปี มีไข้ สง 3 วัน ต่อมามีหอบและซึมลง 1 วัน
                                                                                                      ู
   decreased breath sounds at Rt. Upper lung with dullness on          มีจุดจําเลือดตามตัว Lab: Hb9.8, Hct 30% WBC 28000-
   percussion, rhonchi both lungs, O2 saturation = 85% การ             38000 N 83% band 10% L7% plt 15000, PT 16 PTT 60
   ตรวจทางปฎิบติการใด ช่วยวินจฉัยโรคของผู้ป่วยรายนีได้ ดทสุด
                    ั              ิ                       ี ี         fibrinogen 105 นึกถึงภาวะใดมากทีสุด
    A. Peak expiratory flow rate                                        A. ITP
    B. Portable chest x-ray                                             B. TTP
    C. Spiral CT angiography                                            C. acquired factor VIII deficiency
    D. Arterial blood gas                                               D. DIC
    E. Sputum Gram stain                                                E. vWD
90.ผู้ป่วยชายอายุ 50 ปี no U/D มีประวัติสบบุหรีและกินเหล้ าเป็ น
                                            ู                       95.ผู้ป่วยชายอายุ 38 ปี มีไข้ สงมา 3 d PTA ต่อมาซีมลง ตรวจ
                                                                                                    ู
   ครังคราว เลิกมาแล้ วประมาณ 1 ปี เพราะว่ามีปัญหา ED ตรวจ             ร่างกายพบมี mild pale, no Jaundice, no hepatosplenomegaly,
   เลือด Chol. 210, TG 150, HDL 55, LDL 120 จะวาง                      Petechiae & Purpura all extremities ผล CBC พบมี Hct 28,
   แผนการรักษาอย่างไรต่อไป                                             WBC 28,000, N 95%, L 5%, Band 10%, Plt 40,000
    A. ไม่ทาอะไร --> normal
                  ํ                                                    ตรวจ PBS พบมี Schistocytosis, ตรวจ CoaguloGram พบมี
    B. ให้ จด morning erection                                         PT, aPTT Prolong, Fibrinogen ตํา ,ในผู้ป่วยรายนีน่าจะคิดถึง
    C. ให้ ยา treat ED                                                 โรคใดมากทีสุด
    D. consult psychiatrist                                             A. Vit K def
    E. consult urologist                                                B. primary fibrinolysis
91.ในการป้ องกันการแพร่ระบาดของวัณโรคปอดในบุคลากรทาง                    C. DIC
   สาธารณสุขทีโรงพยาบาล ข้ อใดมีประสิทธิภาพทีสุด                        D. TTP
    A. ให้ ผ้ ป่วย ทีสงสัยวัณโรคสวมหน้ ากาก
              ู                                                         E. Thalassemia with sepsis
    B. ให้ บคลากรทางการแพทย์สวมหน้ ากากป้ องกัน
                ุ                                                   96.ผู้ป่วยหญิงอายุ 18 ปี เลือดออกตามไรฟัน ประจําเดือนมามาก
    C. ติดเครืองปรับอากาศในหอผู้ป่วย                                   ผิดปกติ ไม่มไข้ ไม่มโรคประจําตัว จุดเลือดออกตามตัว แขน ขา
                                                                                     ี      ี
    D. ย้ ายเตียงผู้ป่วยวัณโรคไปอยู่ท้ายวอร์ด                          ตับม้ ามไม่โต Lab: Hb,Hct,WBC normal มี Eo 5% plt 15000
                                                                       APTT 35,PT 12 การวินจฉัยทีเหมาะสมคือ
                                                                                                  ิ


NLE_step_2_2010_NCTMS                                                                                               Page 10
A. Myelo….                                                     100. ผู้ป่วยชายอายุ 18 ปี ถูกทําร้ ายร่างกาย หายใจเหนือย ไอเป็ น
    B. TTP                                                              เลือด เสียงแหบ มี subcutaneous emphysema at neck , air
    C. ITP                                                              hunger management คือ
    D. APDE                                                           A. Intubation
    E. DIC                                                            B. CT larynx
97.ผู้ป่วยหญิงไทย มาโรงพยาบาลด้ วยเรืองกินนํายาล้ างห้ องนํา          C. Tracheotomy
   โจทย์บอกว่าเป็ นด่าง v/s ปกติ (ไม่แน่ใจนะ >>มีแสบบริเวณ            D. barium swallowing
   หน้ าอก และลําคอ) คําถามคือ ควรทําอย่างไร                          E. ABG
    A. กระตุ้นให้ อ้วก                                             101. ผู้ป่วยหญิงอายุ 30 ปี ล้ มหน้ าอกขวากระแทกโต๊ะ 30 นาที
    B. ให้ charcoal                                                     ต่อมามาโรงพยาบาล PE : PR 80 RR 30 BP 110/80 CXR
    C. ให้ NaHco3                                                       : right 4th&5th rib fracture with pneumothorax 15% หลัง
    D. ใส่ NG                                                           ได้ O2 แล้ วทําอะไรต่อ
    E. Early scope                                                    A. closed observe
98.ผู้ป่วยชายอายุ 26 ปี โดนสุนขกัดเป็ นรอยถลอก ไม่มเลือดออก
                                   ั                    ี             B. pain control
   หลังจากทําความสะอาดแผลแล้ ว ควรทําอย่างไร                          C. needle thoracotomy
    A. สังเกตพฤติกรรมสัตว์อย่างเดียว                                  D. thoracotomy
    B. สังเกตพฤติกรรมสัตว์ และให้ ยาปฏิชวนะ   ี                       E. ICD
    C. ให้ วคซีนพิษสุนขบ้ าจนครบ course
                ั          ั                                       102. ผู้ป่วยชายอายุ 26 ปี MCA หน้ าอกกระแทก V/S …………..
    D. ให้ วคซีนพิษสุนขบ้ า และให้ ยาปฏิชีวนะ
                  ั          ั                                          PE: (บรรยายมาเข้ าได้ กบ Beck ‘s triad ) ให้ o2 canula และ
                                                                                                 ั
    E. ให้ วคซีนพิษสุนขบ้ าและ RIG
                    ั          ั                                        IV saline ไปไม่ดีขน จะทําอย่างไรต่อ???
                                                                                          ึ
    F. ให้ tetanus toxoid จนครบ course                                A. ให้ inotrope drug
    G. ให้ tetanus toxoid + TAT จนครบ course                          B. pericardiocentensis
    H. ให้ ยาปฏิชีวนะอย่างเดียว                                    103. ผู้ป่วยชายอายุ 65 ปี ถ่ายปนเลือดเป็ นๆหายๆมา2สัปดาห์ วัน
    I. ให้ tetanus toxoid + TAT จนครบ course ร่วมกับยา                  ละ2ครัง ลักษณะเป็ นอุจจาระปนเลือดสีดาคลําปนมูก ตรวจ
                                                                                                                 ํ
           ปฏิชีวนะ                                                     abdomenปกติ rectal examinationปกติ proctoscopeพบ
99. ผู้ป่วยชายอายุ 40 ปี ถูกมีดบาดขณะปอกผลไม้ 1 ชัวโมงก่อน              อุจจาระสีเหลืองปนมูก ไม่มแผล ไม่มก้อน ไม่พบความ
                                                                                                    ี         ี
      มาโรงพยาบาล แผลกว้ าง 1 cm ลึก 0.5 cm มีเลือดออก                  ผิดปกติอนๆ ควรทําอะไรต่อไป
                                                                                  ื
      เล็กน้อย ได้ ทาการล้ างแผลด้ วยสบู่ และทาเบตาดีนเองจาก
                        ํ
      บ้ าน ผู้ป่วยกลัวว่าจะเป็ นบาดทะยัก จึงมาขอฉีดยากัน             A. bulk forming laxative
      บาดทะยัก ; 4 ปี ก่อนมาโรงพยาบาล ผู้ป่วยเคยถูกตะปูตาทีเท้ า
                                                           ํ          B. fecal occult blood
      ได้ รับการฉีดยากันบาดทะยัก 3 เข็ม ทีโรงพยาบาลทีท่าน             C. barium enema
      ปฏิบติงานอยู่ การปฏิบติทเหมาะสมทีสุด สําหรับผู้ป่วยราย
             ั                    ั ี                                 D. CEA
      นีคือข้ อใด                                                  104. colonoscopyผู้ป่วยชายอายุ 50 ปี มีไข้ ปวดบันเอวขวาร้ าวไป
    A. ไม่จาเป็ นต้ องฉีดยากันบาดทะยัก
                      ํ                                                 หลังและขาหนีบ ปัสสาวะแสบขัด เป็ นๆหายๆ 2 ครัง ไม่เคย
    B. ฉีด Tetanus toxoid 1 เข็ม                                        ถ่ายปัสสาวะเป็ นเลือด ตรวจร่างกายปกติUA : WBC 0-
    C. ฉีด Tetanus toxoid 3 เข็ม                                        1 RBC 10-20 Investigation ทีช่วยในการวินจฉัย ิ
    D. ฉีด Tetanus antitoxin ร่วมกับ Tetanus toxoid 1 เข็ม            A. urine calcium
    E. ฉีด Tetanus antitoxin ร่วมกับ Tetanus toxoid 3 เข็ม            B. IVP
                                                                      C. cystoscope


NLE_step_2_2010_NCTMS                                                                                               Page 11
D. u/s KUB                                                         D. U/S Abdomen
   E. plain KUB                                                       E. BE
105. ผู้ป่วยชายมีประวัติสบบุหรี ปัสสาวะเป็ นเลือดสีแดงตลอดสาย
                          ู                                        110. ผู้ป่วยชายอายุ 70 ปี มาด้ วยปวดท้องทันที 2 ชม. PTA
     ไม่ปวดท้ องน้ อย ไม่ต้องเบ่งเวลา ถ่ายปัสสาวะ investigation?        BP170/80, PR120 bpm irregular pulse PE: Abd soft, not
   A. CT whole abdomen                                                  tender, absent bowel sound CBC ไม่ซีด WBC21,000
   B. Retrograde urethroGram                                            Neutrophil เด่น การวินจฉัยทีเหมาะสมคือ
                                                                                                ิ
   C. Cystoscopy                                                      A. PU perforate
   D. VCUG                                                            B. Acute pancrsatitis
   E. IVP + Cyrtouroscope                                             C. Rupture appendicitis
   F. CystouretheroGram                                               D. Sup. mesenteric a.
   G. Voiding cystouteroGram                                       111. ผู้ป่วยหญิงอายุ 18 ปี MCA PE: Closeก Fx of femur BP
   H. CysouretheroGram +IVP                                             80/50 PR 120 GCS 14 มี Abrasion at right chest
106. ผู้ป่วยชายถ่ายเป็ นเลือดคลํา มานาน PE: ปกติ                        Abdomen : tender at right abdomen หลัง Load NSS แล้ ว
     Procto+Sigmoidoปกติ, -ve bowel habit change ทําอะไรต่อ             V/S ยังไม่ดขน ทําไร
                                                                                     ีึ
   A. Colonoscopy                                                     A. Observe
   B. AngioGram                                                       B. FAST
   C. Abd tapping                                                     C. DPL
107. ไม่มอาการ เจาะ PSA ได้ 3 PR- prostate ปกติ ทําอะไร
           ี                                                          D. CT
   A. TURP                                                            E. Explor.lap
   B. Transrectal U/S                                              112. ผู้หญิงถ่ายอุจจาระเป็ นเลือด บางครังปนเลือดสด PR ปกติ Ix
   C. PR q 1 yr + PSA q 1 yr                                            อะไร
108. ผู้ป่วยชายอายุ 70 yr มีปัสสาวะลําบาก กระปริบกะปรอย               A. Colonoscopy
     ติดขัด ใช้ เวลานาน ไม่พ่งมานานประมาณ 2-3 เดือน, 3
                             ุ                                        B. Sigmoidoscopy
     DPTA ผู้ป่วยเป็ นหวัดคัดจมูก กินยาลดนํามูก, 1 DPTA ผู้ ู         C. ปลาปักเป้ า →Terotoxin
     ป่ วยมาด้ วยปัสสาวะไม่ออก PE : full bladder , PR:             113. ผู้ป่วยชายอายุ 20 ปี ถูกพบหมดสติในอาคารไฟไหม้ PE: PR
     prostate gland enlargement ,rubbery, smooth surface                64 BP 90/70 BP=90/60mmHg RR=12 /min unresponse
     normal anal sphincter tone Mx??                                    to deep pain, black soot in mouth and throat heart and
   A. คาสายสวน 3-5 วัน                                                  lungs unremarkable, no skin burn ส่งตรวจทาง
   B. คาสายสวน 3-5 วัน ให้ ยารักษาต่อมลูกหมากโต                         ห้ องปฏิบติการณ์ใดทีช่วยในการวินจฉัยโรคมากทีสุด
                                                                                  ั                       ิ
   C. คาสายสวน 3-5 วัน และเตรียมผ่าตัดต่อมลูกหมาก                     A. pulse oximetry
   D. Cystostomy                                                      B. serum calcium
   E. ควรส่องกล้ อง cystoscopy เพือหาสาเหตุของอาการ                   C. serum electrolyte
          ปัสสาวะไม่ออก                                               D. serum lactate
109. ผู้ป่วยชายอายุ 50 ปี ปวดลินปี 24 hr ก่อนมา รพ. จากนัน            E. venous blood gas
     ย้ ายมาปวดทีท้ องน้ อย ด้ านขวากดเจ็บ CBC พบ                  114. ผู้ป่วยหญิงอายุ 50 ปี เป็ นโรคเบาหวานมาหลายปี มีอาการไข้
     Leukocytosis ส่งตรวจอะไรเพิมเติม?                                  สูงหนาวสัน ปวดท้ อง มา2วัน PE:tenderness RUQ ให้ ภาพ
   A. Acute abdomen series                                              film abdomen seriesมา(ซึงเราดูไม่ออกง่ะว่าเป็ นอะไร) ถามว่า
   B. CT abdomen                                                        เป็ นโรคอะไร
   C. GI follow through                                               A. gastric perforation


NLE_step_2_2010_NCTMS                                                                                               Page 12
B. ruptured duodenum                                           119. ผู้ป่วยหญิงอายุ 35 ปี ทํางานพิมพ์คอมพิวเตอร์ มีอาการชา
   C. gut obstruction                                                  บริเวณนิวโป้ งชี กลาง ตรวจร่างกายไม่มี hypothenar wasting
   D. emphysematous cholecystitis                                      ไม่มอาการปวดร้ าวไปคอ จงให้ การวินจฉัยผู้ป่วยรายนี
                                                                              ี                                ิ
   E. liver abscess                                                  A. Carpal Tunnel Syndrome
115. ผู้ป่วยหญิงอายุ 45 ปี มีอาการปวดท้ องด้ านบนขวาอย่าง            B. DeQuarvain Syndrome
     เฉียบพลันมา 1 ชัวโมง สักครู่เดียวอาการก็หายไป ตรวจร่าย          C. Polyneuropathy
     กายปกติดี ตรวจท้ อง : no mass กดเจ็บที RUQ Lab CBC              D. C-Spine Sublaxation
     ปกติ , UA ปกติ ถามว่าจะสงตรวจอะไรเพิมเติม                    120. Common Peroneal injury หลัง Stimulation ทําอะไร
   A. U/S                                                            A. Isotonic Exercise
   B. Plain KUB                                                      B. Isometric Ex.
   C. Plain Abdomen                                                  C. Active range of motion
   D. CT abdomen                                                     D. Active…
   E. MRI                                                            E. Passive…
116. ผู้ป่วยหญิงอายุ 55 ปี ได้ รับอุบติเหตุทสะโพกหลังการรักษา
                                     ั      ี                     121. ผู้ป่วยชายอายุ 60 ปี ได้ รับการวินจฉัยเป็ นเบาหวานครังแรก
                                                                                                          ิ
     ด้ วยวิธดึงข้ อ อาการปกติดี ต่อมามีอาการปวดทีสะโพกและ
             ี                                                         สูบบุหรี 2 ซอง/วัน มานาน 40 ปี เมือให้ คาแนะนําให้ ผ้ป่วย
                                                                                                                   ํ           ู
     เดินลําบาก ข้ อใดจะช่วยในการวินจฉัยและวางแผนการรักษา
                                         ิ                             เลิกบุหรี ผู้ป่วยบอกว่ายังไม่พร้ อม ข้ อใดควรปฏิบตมากทีสุด
                                                                                                                         ั ิ
     มากทีสุด                                                        A. ให้ กาลังใจว่าผู้ป่วยสามารถทําได้
                                                                                  ํ
   A. Arthocentesis
                                                                     B. บอกข้ อเสียของการสูบบุหรี
   B. Film boht hip
                                                                     C. แนะนําวิธการเลิกบุหรีทีเหมาะสม
                                                                                       ี
   C. CT
   D. MRI                                                            D. บอกข้ อดีข้อเสียของการเลิกบุหรีแต่ละวิธี
   E. Bone scan                                                      E. บอกให้ มารับคําแนะนําได้ เสมอเมือต้ องการ
117. ผู้ป่วยชายอายุ 3 ปี แม่พามาตรวจด้ วยเรืองก้ อนในท้ องข้ าง   122. ผู้ป่วยชายอายุ 45 ปี เป็ น AIDs มา 5 ปี 2 wkก่อนมีไข้
     ซ้ าย ถามว่าจะ investigate อะไร                                   หอบ พบว่าเป็ น PCP รักษามา 1wk อาการหอบดีขน           ึ
   A. film abdomen                                                     ตอนีมีสบสน disoriented to time-place-person นอนไม่หลับ
                                                                                ั
   B. u/s                                                              กระสับกระส่าย ให้ วนจฉัย
                                                                                           ิ ิ
   C. CT                                                             A. acute psychosis
   D. MRI                                                            B. delirium
118. ผู้ป่วยชายอายุ 70 ปี no underlying. ไข้ หนาวสัน 2 วัน ปวด       C. AID dementia
     ท้ องบริเวณใต้ ชายโครงขวา PE: BT 38 c mild icteric sclera,      D. Pseudodemantia
     not pale, mild abdominal distendion, liver just palpable,       E. schizophrenia
     tenderness on RUQ percussion จงให้ การวินจฉัยทีเป็ นไป
                                                 ิ                123. ผู้ป่วยชายอายุ 75 ปี เป็ นทหารอยู่ภาคใต้ ปราบปราม
     ได้ มากทีสุด                                                      ผู้กอการร้ าย โดนยิง เข้ ารับการรักษาด้ วยการผ่าตัด หลังผ่าตัด
                                                                            ่
   A. liver abscess                                                    แผลหายเป็ นปกติ แต่มอาการกลัวเสียงดัง นอนไม่หลับ ไม่
                                                                                                ี
   B. cholecystitis                                                    สนใจสิงแวดล้ อมรอบตัว ข้ อใดคือ diagnosis
   C. cholangitis                                                    A. Acute stress response
   D. gallstone pancreatitis                                         B. Post traumatic Stress Disorder
   E. cholangiocarcinoma                                             C. Normal reaction
                                                                     D. Brief Psychotic Disorder

NLE_step_2_2010_NCTMS                                                                                                 Page 13
E. Adjustment disorser                                             128. ผู้ป่วยชายอายุ 79 yr หลงลืมมา5ปี จําลูกไม่ได้ โวยวาย ไม่
124. ผู้ป่วยชายอายุ 45 ปี ดืมเหล้ าขาววันละ 1 ขวด มา 25 ปี ถ่าย            นอน เห็นภาพหลอนเป็ น สามีทตาย PE: disoriented
                                                                                                            ี
     เหลวมา 3-4 ครัง/d เป็ นเวลา 2 วัน มาด้ วยอาการซึมลง                   impaired short&long memorycalculate&language deficit
     สับสน จําญาติไม่ได้ มา 3 วัน ระหว่างนีกินเหล้ าตลอด ตรวจ              Tx??
     ร่างกาย V/S ปกติ PE: stupor, Erythematous velvety                   A. Vit.E
     hyperpigmented Desquamous scaling plaque at face neck               B. Gingko
     and both forearms การวินจฉัยทีเหมาะสมคือ
                                  ิ                                      C. Donepezil
   A. Delirium tremens                                                   D. Diazepam
   B. Subdural hematoma                                                  E. Haloperidol
   C. Pellagra                                                        129. ผู้ป่วยชายอายุ 14 ปี บิดามาพบแพทย์ เคยโดนครูเรียกบ่อยๆ
   D. Electrolyte imbalance                                                ลูกโดดเรียน สูบบุหรี วิวาท เด็กไม่ทราบสาเหตุทมาพบแพทย์
                                                                                                                           ี
   E. Wernicke's encephalopathy                                            Mx???
   F. Alcohol intoxication                                               A. ถามความต้ องการทีแท้ จริงของเด็ก
   G. Hepatic encephalopathy                                             B. ให้ บดาบอกเด็กตามจริง
                                                                                    ิ
125. ผู้ป่วยหญิงอายุ 79 ปี แข็งแรงดี หลงลืมมา 8 เดือน จําไม่ได้          C. สร้ างความสัมพันธ์กบเด็ก  ั
     เก็บเงินไว้ ทไหน ทําอาหารลืมปิ ดเตา เคยคิดว่าอยู่บ้านคนอืน
                  ี                                                      D. ปิ ดเป็ นความลับ
     ทังทีอยู่บ้านของตน มีหงุดหงิด นอนหลับๆตืนๆ PE: normal               E. ส่งปัสสาวะตรวจ
     Mental status: impair orentiated to time, impair short and       130. ผู้ป่วยชาย smoke 20 มวน/วัน U/D DM แพทย์แนะนําหยุด
     long term memory, impair calculate and language ถาม                   บุหรี แต่ผ้ ูป่วยไม่พร้ อม แพทย์ควร
     diagnosis ?                                                         A. บอกข้ อเสียบุหรี
   A. normal age change                                                  B. บอกวิธการเลิกี
   B. mild cognitive dysfunction                                         C. สามารถกลับมาปรึกษาใหม่ได้
   C. dementia                                                           D. บอกข้ อดีข้อเสียของบุหรีแต่ละวิธี
   D. derilium                                                        131. ผู้ป่วยหญิงอายุ 22ปี มีประวัติซึมเศร้ ามา 1ปี 3สัปดาห์กอน
                                                                                                                                  ่
   E. depression                                                           คุยเก่ง ซือของมาก คุยกับเพือนว่ามีแฟนเป็ นนักร้ องดัง โทรจิต
126. ผู้ป่วยชายอายุ 35 ปี ผ่าตัดไส้ ติง หลังผ่าตัด 3 วัน เห็นสาย           คุยกันทุกวัน ให้ ยา อะไร
     นําเกลือเป็ นงู มีประวัติกนเหล้ ามา 10 ปี จงวินจฉัย...
                                ิ                       ิ                A. Lithium + Valpoic
   A. Delirium tremens                                                   B. Lithium + Fluroxetine
   B. Delusion disorder                                                  C. Valpoic + Hadol
   C. Schizophrenia                                                      D. Hadol + Amitryptiline
   D. Hepatic encephalopathy                                             E. Hadol +…
   E. Hypochondriasis                                                 132. ผู้ป่วยชาย มาด้ วยชัก ไม่เคยชักมาก่อน กินเหล้ า มา 20 ปี
127. ผู้ป่วยชายอายุ 65 ปี กินเหล้ าทุกวัน มาadmit กระดูกข้ อเท้ า          สามวันที ผ่านมากินเหล้ าทุกวัน ชักมา 15 นาทีแล้ วมารพ.
     หัก 3 วัน เพ้ อคลัง ปี น เตียง คิดว่าจะมีคนทําร้ าย จะให้ อะไร        มาถึงก็ยังชักอยู่ Mx??
   A. midazolam                                                          A. Diazepam IV
   B. diazepam                                                           B. ETtube
   C. Haloperidol                                                        C. Phenytoin
   D. Clonazepam
   E. Chlorpomazine


NLE_step_2_2010_NCTMS                                                                                                   Page 14
Nle step 2_2553
Nle step 2_2553

More Related Content

What's hot

Adult Nursing II มโนทัศน์การพยาบาลผู้ใหญ่ที่เจ็บป่วยซับซ้อนในระยะเฉียบพลันแล...
Adult Nursing II  มโนทัศน์การพยาบาลผู้ใหญ่ที่เจ็บป่วยซับซ้อนในระยะเฉียบพลันแล...Adult Nursing II  มโนทัศน์การพยาบาลผู้ใหญ่ที่เจ็บป่วยซับซ้อนในระยะเฉียบพลันแล...
Adult Nursing II มโนทัศน์การพยาบาลผู้ใหญ่ที่เจ็บป่วยซับซ้อนในระยะเฉียบพลันแล...Chutchavarn Wongsaree
 
Ch 8 basic emergency medical service and triage
Ch 8 basic emergency medical service and triageCh 8 basic emergency medical service and triage
Ch 8 basic emergency medical service and triagefreelance
 
Risk management กับ CQI - Suradet Sriangkoon
Risk management กับ CQI - Suradet SriangkoonRisk management กับ CQI - Suradet Sriangkoon
Risk management กับ CQI - Suradet SriangkoonSuradet Sriangkoon
 
PALS manual 2009
PALS manual 2009PALS manual 2009
PALS manual 2009taem
 
12 กิจกรรมทบทวน
12 กิจกรรมทบทวน12 กิจกรรมทบทวน
12 กิจกรรมทบทวนSuradet Sriangkoon
 
27การตรวจครรภ์
27การตรวจครรภ์27การตรวจครรภ์
27การตรวจครรภ์Papawee Laonoi
 
Trauma Initial assessment and Resuscitation
Trauma Initial assessment and ResuscitationTrauma Initial assessment and Resuscitation
Trauma Initial assessment and ResuscitationNarenthorn EMS Center
 
กลไกการคลอดปกติ 8 ขั้นตอน
กลไกการคลอดปกติ 8 ขั้นตอนกลไกการคลอดปกติ 8 ขั้นตอน
กลไกการคลอดปกติ 8 ขั้นตอนda priyada
 
การวัดความดันในหลอดเลือดดำกลาง Yui
การวัดความดันในหลอดเลือดดำกลาง Yuiการวัดความดันในหลอดเลือดดำกลาง Yui
การวัดความดันในหลอดเลือดดำกลาง Yuipiyarat wongnai
 
การส่งเสริมการขับถ่ายอุจจาระ 57
การส่งเสริมการขับถ่ายอุจจาระ 57การส่งเสริมการขับถ่ายอุจจาระ 57
การส่งเสริมการขับถ่ายอุจจาระ 57Sirinoot Jantharangkul
 
12 กิจกรรมทบทวนภาคปฏิบัติ - Suradet Sriangkoon
12 กิจกรรมทบทวนภาคปฏิบัติ  - Suradet Sriangkoon12 กิจกรรมทบทวนภาคปฏิบัติ  - Suradet Sriangkoon
12 กิจกรรมทบทวนภาคปฏิบัติ - Suradet SriangkoonSuradet Sriangkoon
 
Central venous pressure (cvp)
Central venous pressure (cvp)Central venous pressure (cvp)
Central venous pressure (cvp)piyarat wongnai
 
การพยาบาลผู้ป่วยผ่าตัดมะเร็งเต้านม
การพยาบาลผู้ป่วยผ่าตัดมะเร็งเต้านม  การพยาบาลผู้ป่วยผ่าตัดมะเร็งเต้านม
การพยาบาลผู้ป่วยผ่าตัดมะเร็งเต้านม Parinya Damrongpokkapun
 
Clinical Tracer DHF.pptx
Clinical Tracer DHF.pptxClinical Tracer DHF.pptx
Clinical Tracer DHF.pptxphalita2
 

What's hot (20)

2016 Respiratory Assessment
2016 Respiratory Assessment2016 Respiratory Assessment
2016 Respiratory Assessment
 
Adult Nursing II มโนทัศน์การพยาบาลผู้ใหญ่ที่เจ็บป่วยซับซ้อนในระยะเฉียบพลันแล...
Adult Nursing II  มโนทัศน์การพยาบาลผู้ใหญ่ที่เจ็บป่วยซับซ้อนในระยะเฉียบพลันแล...Adult Nursing II  มโนทัศน์การพยาบาลผู้ใหญ่ที่เจ็บป่วยซับซ้อนในระยะเฉียบพลันแล...
Adult Nursing II มโนทัศน์การพยาบาลผู้ใหญ่ที่เจ็บป่วยซับซ้อนในระยะเฉียบพลันแล...
 
Ch 8 basic emergency medical service and triage
Ch 8 basic emergency medical service and triageCh 8 basic emergency medical service and triage
Ch 8 basic emergency medical service and triage
 
Risk management กับ CQI - Suradet Sriangkoon
Risk management กับ CQI - Suradet SriangkoonRisk management กับ CQI - Suradet Sriangkoon
Risk management กับ CQI - Suradet Sriangkoon
 
PALS manual 2009
PALS manual 2009PALS manual 2009
PALS manual 2009
 
12 กิจกรรมทบทวน
12 กิจกรรมทบทวน12 กิจกรรมทบทวน
12 กิจกรรมทบทวน
 
27การตรวจครรภ์
27การตรวจครรภ์27การตรวจครรภ์
27การตรวจครรภ์
 
Trauma Initial assessment and Resuscitation
Trauma Initial assessment and ResuscitationTrauma Initial assessment and Resuscitation
Trauma Initial assessment and Resuscitation
 
Example osce
Example osceExample osce
Example osce
 
กลไกการคลอดปกติ 8 ขั้นตอน
กลไกการคลอดปกติ 8 ขั้นตอนกลไกการคลอดปกติ 8 ขั้นตอน
กลไกการคลอดปกติ 8 ขั้นตอน
 
การวัดความดันในหลอดเลือดดำกลาง Yui
การวัดความดันในหลอดเลือดดำกลาง Yuiการวัดความดันในหลอดเลือดดำกลาง Yui
การวัดความดันในหลอดเลือดดำกลาง Yui
 
จำแนกประเภท
จำแนกประเภทจำแนกประเภท
จำแนกประเภท
 
Warning sign
Warning signWarning sign
Warning sign
 
การส่งเสริมการขับถ่ายอุจจาระ 57
การส่งเสริมการขับถ่ายอุจจาระ 57การส่งเสริมการขับถ่ายอุจจาระ 57
การส่งเสริมการขับถ่ายอุจจาระ 57
 
12 กิจกรรมทบทวนภาคปฏิบัติ - Suradet Sriangkoon
12 กิจกรรมทบทวนภาคปฏิบัติ  - Suradet Sriangkoon12 กิจกรรมทบทวนภาคปฏิบัติ  - Suradet Sriangkoon
12 กิจกรรมทบทวนภาคปฏิบัติ - Suradet Sriangkoon
 
เกณฑ์การวัดสมรรถนะระดับปฏิบัติการ
เกณฑ์การวัดสมรรถนะระดับปฏิบัติการเกณฑ์การวัดสมรรถนะระดับปฏิบัติการ
เกณฑ์การวัดสมรรถนะระดับปฏิบัติการ
 
Kanniga 31 jan
Kanniga 31 janKanniga 31 jan
Kanniga 31 jan
 
Central venous pressure (cvp)
Central venous pressure (cvp)Central venous pressure (cvp)
Central venous pressure (cvp)
 
การพยาบาลผู้ป่วยผ่าตัดมะเร็งเต้านม
การพยาบาลผู้ป่วยผ่าตัดมะเร็งเต้านม  การพยาบาลผู้ป่วยผ่าตัดมะเร็งเต้านม
การพยาบาลผู้ป่วยผ่าตัดมะเร็งเต้านม
 
Clinical Tracer DHF.pptx
Clinical Tracer DHF.pptxClinical Tracer DHF.pptx
Clinical Tracer DHF.pptx
 

Similar to Nle step 2_2553

Compre si 2010 l
Compre si 2010 lCompre si 2010 l
Compre si 2010 lvora kun
 
สอบ-ศรว-มีนาคม-2551
สอบ-ศรว-มีนาคม-2551สอบ-ศรว-มีนาคม-2551
สอบ-ศรว-มีนาคม-2551rookiess
 
2007821172158 466 6438_1
2007821172158 466 6438_12007821172158 466 6438_1
2007821172158 466 6438_1New Srsn
 
Exercise national license_part_ii_march_2009_2
Exercise national license_part_ii_march_2009_2Exercise national license_part_ii_march_2009_2
Exercise national license_part_ii_march_2009_2Loveis1able Khumpuangdee
 
5696 6770-1-pb
5696 6770-1-pb5696 6770-1-pb
5696 6770-1-pbNew Srsn
 
Gynecologic Malignancy
Gynecologic MalignancyGynecologic Malignancy
Gynecologic Malignancyanucha98
 
Ped emergency final to ems พญ.ศิราภรณ์ สวัสดิวร
Ped emergency final to ems พญ.ศิราภรณ์ สวัสดิวรPed emergency final to ems พญ.ศิราภรณ์ สวัสดิวร
Ped emergency final to ems พญ.ศิราภรณ์ สวัสดิวรtaem
 
Skill manual removal of placenta
Skill manual removal of placentaSkill manual removal of placenta
Skill manual removal of placentaHummd Mdhum
 
รังสีวินิจฉัยและเวชศาสตร์นิวเคลียร์
รังสีวินิจฉัยและเวชศาสตร์นิวเคลียร์รังสีวินิจฉัยและเวชศาสตร์นิวเคลียร์
รังสีวินิจฉัยและเวชศาสตร์นิวเคลียร์techno UCH
 
แนวทางการดูแลรักษาลิ้มเลือดอุดตันในปอด ในผูปวยฉุกเฉิน
แนวทางการดูแลรักษาลิ้มเลือดอุดตันในปอด ในผูปวยฉุกเฉินแนวทางการดูแลรักษาลิ้มเลือดอุดตันในปอด ในผูปวยฉุกเฉิน
แนวทางการดูแลรักษาลิ้มเลือดอุดตันในปอด ในผูปวยฉุกเฉินSociety of Thai Emergency Physicians
 

Similar to Nle step 2_2553 (18)

Compre si 2010 l
Compre si 2010 lCompre si 2010 l
Compre si 2010 l
 
สอบ-ศรว-มีนาคม-2551
สอบ-ศรว-มีนาคม-2551สอบ-ศรว-มีนาคม-2551
สอบ-ศรว-มีนาคม-2551
 
Hiv guideline 2557
Hiv guideline 2557Hiv guideline 2557
Hiv guideline 2557
 
For extern
For externFor extern
For extern
 
National test _2553_TU
National test _2553_TUNational test _2553_TU
National test _2553_TU
 
2007821172158 466 6438_1
2007821172158 466 6438_12007821172158 466 6438_1
2007821172158 466 6438_1
 
Buddhm202
Buddhm202Buddhm202
Buddhm202
 
Nl part ii march 2009
Nl part ii march 2009Nl part ii march 2009
Nl part ii march 2009
 
Nl part ii march 2009
Nl part ii march 2009Nl part ii march 2009
Nl part ii march 2009
 
Exercise national license_part_ii_march_2009_2
Exercise national license_part_ii_march_2009_2Exercise national license_part_ii_march_2009_2
Exercise national license_part_ii_march_2009_2
 
Bleeding in first half
Bleeding in first halfBleeding in first half
Bleeding in first half
 
5696 6770-1-pb
5696 6770-1-pb5696 6770-1-pb
5696 6770-1-pb
 
Gynecologic Malignancy
Gynecologic MalignancyGynecologic Malignancy
Gynecologic Malignancy
 
Ped emergency final to ems พญ.ศิราภรณ์ สวัสดิวร
Ped emergency final to ems พญ.ศิราภรณ์ สวัสดิวรPed emergency final to ems พญ.ศิราภรณ์ สวัสดิวร
Ped emergency final to ems พญ.ศิราภรณ์ สวัสดิวร
 
Skill manual removal of placenta
Skill manual removal of placentaSkill manual removal of placenta
Skill manual removal of placenta
 
Cpg std aug 2011
Cpg std aug 2011Cpg std aug 2011
Cpg std aug 2011
 
รังสีวินิจฉัยและเวชศาสตร์นิวเคลียร์
รังสีวินิจฉัยและเวชศาสตร์นิวเคลียร์รังสีวินิจฉัยและเวชศาสตร์นิวเคลียร์
รังสีวินิจฉัยและเวชศาสตร์นิวเคลียร์
 
แนวทางการดูแลรักษาลิ้มเลือดอุดตันในปอด ในผูปวยฉุกเฉิน
แนวทางการดูแลรักษาลิ้มเลือดอุดตันในปอด ในผูปวยฉุกเฉินแนวทางการดูแลรักษาลิ้มเลือดอุดตันในปอด ในผูปวยฉุกเฉิน
แนวทางการดูแลรักษาลิ้มเลือดอุดตันในปอด ในผูปวยฉุกเฉิน
 

More from Loveis1able Khumpuangdee (20)

Rollup01
Rollup01Rollup01
Rollup01
 
Protec
ProtecProtec
Protec
 
Factsheet hfm
Factsheet hfmFactsheet hfm
Factsheet hfm
 
Factsheet
FactsheetFactsheet
Factsheet
 
Eidnotebook54
Eidnotebook54Eidnotebook54
Eidnotebook54
 
Data l3 148
Data l3 148Data l3 148
Data l3 148
 
Data l3 147
Data l3 147Data l3 147
Data l3 147
 
Data l3 127
Data l3 127Data l3 127
Data l3 127
 
Data l3 126
Data l3 126Data l3 126
Data l3 126
 
Data l3 113
Data l3 113Data l3 113
Data l3 113
 
Data l3 112
Data l3 112Data l3 112
Data l3 112
 
Data l3 92
Data l3 92Data l3 92
Data l3 92
 
Data l3 89
Data l3 89Data l3 89
Data l3 89
 
Data l2 80
Data l2 80Data l2 80
Data l2 80
 
Hfm reccomment10072555
Hfm reccomment10072555Hfm reccomment10072555
Hfm reccomment10072555
 
Hfm work2550
Hfm work2550Hfm work2550
Hfm work2550
 
Factsheet hfm
Factsheet hfmFactsheet hfm
Factsheet hfm
 
Publichealth
PublichealthPublichealth
Publichealth
 
แนวทางการดาเน ํ นงานป ิ องก ้ นควบค ั มการระบาดของโรคม ุ ือ เท้า ปาก สําหรบแพ...
แนวทางการดาเน ํ นงานป ิ องก ้ นควบค ั มการระบาดของโรคม ุ ือ เท้า ปาก สําหรบแพ...แนวทางการดาเน ํ นงานป ิ องก ้ นควบค ั มการระบาดของโรคม ุ ือ เท้า ปาก สําหรบแพ...
แนวทางการดาเน ํ นงานป ิ องก ้ นควบค ั มการระบาดของโรคม ุ ือ เท้า ปาก สําหรบแพ...
 
hand foot mouth
hand foot mouthhand foot mouth
hand foot mouth
 

Nle step 2_2553

  • 1. สหพั น ธ์ นิ สิ ตนั กศึ กษาแพทย์ แห่ งประเทศไทย (สพท.) National Council of Thai Medical Students (NCTMS) วิทยาลัยแพทยศาสตร์กรุงเทพมหานครและวชิรพยาบาล คณะแพทยศาสตร์ มหาวิทยาลัยขอนแก่น คณะแพทยศาสตร์ จุฬาลงกรณ์มหาวิทยาลัย คณะแพทยศาสตร์ มหาวิทยาลัยเชียงใหม่ คณะแพทยศาสตร์ มหาวิทยาลัยธรรมศาสตร์ คณะแพทยศาสตร์ มหาวิทยาลัยนราธิวาสราชนครินทร์ คณะแพทยศาสตร์ มหาวิทยาลัยนเรศวร คณะแพทยศาสตร์ มหาวิทยาลัยบูรพา คณะแพทยศาสตร์พระบรมราชชนก วิทยาลัยแพทยศาสตร์พระมงกุฎเกล้า คณะแพทยศาสตร์ มหาวิทยาลัยมหาสารคาม วิทยาลัยแพทยศาสตร์ มหาวิทยาลัยรังสิต คณะแพทยศาสตร์โรงพยาบาลรามาธิบดี สํานักวิชาแพทยศาสตร์ มหาวิทยาลัยวลัยลักษณ์ คณะแพทยศาสตร์ มหาวิทยาลัยศรีนครินทรวิโรฒ คณะแพทยศาสตร์ศรราชพยาบาล ิิ คณะแพทยศาสตร์ มหาวิทยาลัยสงขลานครินทร์ สํานักวิชาแพทยศาสตร์ มหาวิทยาลัยเทคโนโลยีสุรนารี วิทยาลัยแพทยศาสตร์และสาธารณสุข มหาวิทยาลัยอุบลราชธานี ๓๑๗/๕ ถนนราชวิถี แขวงทุงพญาไท เขตราชเทวี กรุงเทพมหานคร ๑๐๔๐๐ โทรสาร ๐-๒๓๕๔-๗๗๕๐ ต่อ ๑๑๒ ่ Exercise for National License Part II CLINICAL SCIENCE : March 2010 By NCTMS2010 แนวข้อสอบชุดนีเป็ นการรวบรวมแนวข้อสอบ ทีส่งมาจากโรงเรียนแพทย์ 2 แห่ง ได้แก่ BM คณะแพทยศาสตร์วชิรพยาบาล มหาวิทยาลัยกรุงเทพมหานคร (วิทยาลัยแพทยศาสตร์กรุงเทพมหานครและวชิรพยาบาล) TU คณะแพทยศาสตร์ มหาวิทยาลัยธรรมศาสตร์ รวบรวมและเรียบเรียงโดย สโมสรนิสิตคณะแพทยศาสตร์ จุฬาลงกรณ์มหาวิทยาลัย ตรวจทานโดย ฝ่ ายวิชาการ สหพันธ์นิสิตนักศึกษาแพทย์แห่ งประเทศไทย รุ่นที 20 หากมีขอผิดพลาดประการใด ขออภัยมาไว้ ณ ทีนี ้ แนะนําหรือติชมได้ที www.nctms.in.th/webboard หมายเหตุ : ข้อสอบฉบับนีเรียบเรียงจาก 1. ผู้ป่วยหญิงอายุ 28 ปี GA10 WK HBsAG +ve ตัวเดียวเท่านัน ต้ องแนะนําอย่างไร A. termination B. C/S at term C. avoid breast feeding D. vaccine at birth E. immunoglobulin at birth 2. ผู้ป่วยหญิงอายุ 25 ปี G1 GA 20 wk. U/S พบ anencephaly การปฏิบตใดเหมาะสมทีสุด ั ิ A. ตรวจเลือดแม่ดู AFP B. ตรวจเลือดแม่ดู triple test C. ตรวจ MRI เพือยืนยันการวินจฉัย ิ NLE_step_2_2010_NCTMS ซําซ้อนกัน จึ งมีเนือหาส่วนใหญ่ Page 1
  • 2. D. ตรวจติดตาม U/S อย่างใกล้ ชิด 8. ผู้ป่วยหญิงอายุ 65 ปี หมดประจําเดือนมา 15 ปี ไม่ได้ E. แนะนํายุติการตังครรภ์ รับประทานฮอร์โมนทดแทน มารพ.ด้ วย AUB 5 วัน จะ 3. แม่อายุ 40 yr แต่งงานมา 10 yr ไม่ได้ คมกําเนิด GA 22wk ุ Investigate อะไร chromosome study 47+xy trisomy 21 จะแนะนําอย่างไร A. Fractional curragetage A. บอกแม่ว่าเป็ นทารกเพศชาย และมีความเสียงในการเกิด B. Endomethrial biopsy ความผิดปกติ 9. ผู้ป่วยหญิงอายุ 25 ปี RlQ pain 2 day PTA LMP 7 days ago B. บอกแม่ว่าผิดปกติ และวางแผนร่วมกันรักษา ,PE tender at RLQ no guarding , PV tender at uterus C. U/S หาความผิดปกติ buldging at anexa การวินจฉัยทีถูกต้ องคือ ิ 4. หญิงไทยคู่อายุ 24 ปี GA 32 wks มีนาไหลออกจากช่องคลอด ํ A. PID 3 hr ก่อนมาโรงพยาบาล ไม่เจ็บท้อง speculum exam พบนํา 5 B. appendiditis ml, nitrazine test positive ต้ องทําอะไร 10.ผู้ป่วยหญิงอายุ 30 ปี แต่งงานมาแล้ ว 6 ปี ไม่เคยคุมกําเนิด A. oxytocin iv ปัจจุบนไม่เคยตังครรภ์และมีลูก มีอาการปวดประจําเดือนพอทน ั B. terbutaline iv ได้ ไม่ต้องหยุดงาน ตรวจร่างกายพบnormal uterine , mass C. C/S cystic content at Rt. adnexa 3x4 cm. tender , cul-du-sac D. dexamethasone im roughly การวินจฉัยทีเหมาะสมคือ ิ E. prostaglandin A. Endrometriosis Cyst 5. ผู้ป่วยหญิงไทยคู่อายุ 24 ปี G?P? GA 3rd trimesters เจาะถุง B. Functional Ovarian cyst นําครําได้ เป็ นนําใส FHR 80-90 bpm PV ซําพบ pulsatile C. Ovarian Tumor mass การวินจฉัยทีเหมาะสมคือ ิ D. TOA A. Vasa previa E. Appendiceal abscess B. Prolapsed cord 11.ผู้ป่วยหญิงอายุ 25 ปี คัน ตกขาว ให้ รปมาKOH น่าจะเป็ นพวก ู C. Placenta previa yeast รักษายังไง? 6. ผู้ป่วยหญิงอายุ 23 ปี GA 39 Wk พบว่าลูกไม่ดิน 24 Hr. PE: A. metronidazole oral Fundal Height ¾ > Umbilicus, FHR 140/min ข้ อใดเป็ นแนว B. norfloxacin oral ทางการรักษาทีเหมาะทีสุด ในผู้ป่วยรายนี C. metronidazole suppo A. Reassure and F/U next day D. clotrimazole B. Biophysical profile 12.ตามมาตรา 276 ประมวลกฎหมายอาญาข้ อใดทีอาจมีความผิด C. Oxytocin stress test ฐานข่มขืนกระทําชําเรา(Rape) D. Induction of labor A. ผู้ชายใช้ อวัยวะเพศเทียมเพศชายใส่เข้ าช่องปากหญิงอืน E. C/S B. ผู้ชายใช้ อวัยวะเพศเทียมเพศชายใส่เข้ าช่องปากชายอืน 7. ผู้ป่วยหญิงอายุ 30 ปี ยังไม่มบุตรมาตรวจร่างกาย ไม่มี ี C. ผู้หญิงใช้ อวัยวะเพศเทียมเพศชายใส่เข้ าช่องปากชายอืน เลือดออกผิดปกติ คลํา Uetrus 12 wk sizeU/S Uterus โต 11 D. ผู้ชายใช้ อวัยวะเพศของตนเองถูไถบริเวณทวารหนักของ cm,Globular shape,Multiple myoma ก้ อนใหญ่สด 2×2 ุ หญิงอืน anterior,2×3 posterior management อย่างไร E. ผู้หญิงใช้ อวัยวะเพศชายเทียมสอดใส่เข้ าช่องทวารหนักของ A. สังเกตอาการและ F/U 3 mo หญิงอืน B. GnRH releasing hormone 13.หญิงปัญญาอ่อน ญาติพาแจ้ งความถูกข่มขืนมา3วันก่อนมา C. Uterine artery embolization โรงพยาบาล ตรวจร่างกายพบว่ามีอสุจิในช่องคลอด acid D. Myomectomy phosphatase positive มีkissing bite มีแผลถลอกใหม่ทช่อง ี E. Hysterectomy NLE_step_2_2010_NCTMS Page 2
  • 3. คลอด และ รอยฉีกขาดใหม่ทางด้ านหลังของช่องคลอด ถามว่า กลับมาทํางานได้ ตามปกติ ตํารวจตังข้ อหาและศาลสังฟ้ องฝ่ าย อะไรทีทําให้ แพทย์ลงความเห็นว่า ถูกกระทําชําเรามา 3วัน ชายว่าทําร้ ายร่างกายจนสาหัส โดยใช้ เหตุผลข้ อใดตามประมวล A. พบอสุจในช่องคลอด ิ กฎหมายอาญา มาตรา 276 B. Acid phosphatase positive A. ตามองไม่เห็น C. พบkissing bite B. แผลบริเวณใบหน้ า D. มีรอยถลอกทีช่องคลอด C. แผลฟกชําทีท้ องน้ อย E. มีรอยฉีกขาดทีทางด้ านหลังของช่องคลอด D. เลือดออกทางช่องคลอด 14.ผู้ป่วยหญิงไทยอายุ 15 ปี เคยมีระดูมาแล้ ว มีอาการขาดระดูมา E. รอยชําทีเบ้ าตา 2 สัปดาห์ ประวัตถูกข่มขืนมา 5 ชัวโมง ตรวจร่างกายพบรอย ิ F. บาดแผลขอบเรียบ 10 ซม. kiss mark ทีคอ มีรอยฟกชําทีบริเวณท้ องน้ อย ตรวจภายในพบ G. ผล pathology ได้ Chrorionic villi 18.ผู้ป่วยชายไทยอายุ 80 ปี U/D DM, HT, CKD, IHD อาศัยอยู่ รอยฟกชํารอบอวัยวะเพศ และพบมูกสีขาวใสในช่องคลอด ลง กับภรรยาอายุ 75 ปี เพียงลําพัง ปฏิเสธการย้ ายไปอยู่กบ ั ความเห็นว่าน่าจะถูกกระทําชําเราภายในเวลา 3 วัน จะ ลูกหลานเพราะผู้ป่วยอยากทําอะไรด้ วยตนเอง ถึงแม้ ว่าในขณะนี Management อย่างไร จะให้ ภรรยาช่วยจัดยาให้ ช่วยพยุงเดินช่วง 2-3 เดือนมานีผู้ป่วย A. ส่ง acid phosphatase มีอาการเบือหน่ายไม่อยากทําอะไร นําหนักลด เบืออาหาร บ่นว่า B. Urine pregnancy test อยากตายอยู่บ่อย ๆ มีอยู่วนหนึงผู้ป่วยเกิดหัวใจวายกะทันหัน ั C. ให้ Norfloxacin ญาตินาส่งรพ. หลังจากกู้ชีวต ฟื นฟูสภาพ ล้ างไตและกลับบ้ าน ํ ิ D. ให้ Levonorgestrel ได้ แต่ยงต้ องมาล้ างไต 3/สัปดาห์ ครังนีผู้ป่วยขอให้ แพทย์หยุด ั E. แนะนําให้ กลับมารักษาหากยังขาดระดูอยู่ ล้ างไต และขอให้ ตนจากไปอย่างสงบ แพทย์ควรทําอย่างไรกับ 15.ศพผู้ชายนอนเสียชีวตอยู่ข้างบาทวิถี ตรวจพบแผลขอบไม่เรียบที ิ ผู้ป่วยรายนี ท้ ายทอย, กะโหลกศีรษะด้ านหลังแตกยาวประมาณ 5 cm. มี A. ทําตามทีผู้ป่วยต้ องการ และให้ เซ็นปฏิเสธการรักษา brain contusion ด้ านหน้ ามากกว่าด้ านหลัง ถามสาเหตุการ B. แจ้ งญาติเกียวกับความต้ องการของผู้ป่วยและเซ็นรับรอง เสียชีวต ิ ความต้ องการ A. ถูกตีด้วยของแข็งทีท้ ายทอย C. แจ้ งลูกให้ รับรู้ความต้ องการของผู้ป่วยและตัดสินใจ B. ล้ มท้ ายทอยกระแทกขอบบาทวิถี D. แจ้ งภรรยาเกียวกับความต้ องการและตัดสินใจ C. ถูกตีด้วยของแข็งทีหน้ าผาก E. พูดคุยกับผู้ป่วยและรักษาอาการซึม D. ล้ มหน้ าผากกระแทกขอบบาทวิถี 19.ผู้ป่วยเด็กชายอายุ 1 ปี มีไข้ ซีด อย่างรวดเร็ว PE: mild pale, E. ถูกตีทท้ายทอยและหน้ าผาก ี moderate jaundice, hepatosplenomegaly Lab: Hct 21% 16.ศาลยกฟ้ องโจทก์คดี จําเลยโดนยิงด้ วยปื นลูกโดด ถามว่า WBC 13800 Plt 245000 , PBS: hypochromic microcytic บาดแผลเป็ นอย่างไร RBC, marked anisocytosis + poikilocytosis จะส่งตรวจอะไร A. กลม มีรอยไหม้ เพือช่วยในการวินจฉัย ิ B. รี ถลอกมีรอยไหม้ A. G6PD screening C. Star shape B. Hb typing 17.การตรวจร่างกายหญิงรายหนึงทีถูกทําร้ ายร่างกายโดยฝ่ ายชาย C. Coomb's test ตรวจร่างกายพบว่าตาขวามองไม่เห็นเนืองจากลืมตาไม่ขน มีรอย ึ D. serum ferritin ฟกชําทีเบ้ าตา มีบาดแผลขอบเรียบบริเวณใบหน้ าข้ างขวาขนาด E. OF test 10 cm. มีแผลฟกชําบริเวณท้ องน้ อย ตรวจพบเลือดออกทาง 20.ผู้ป่วยเด็กชายอายุ 7 ปี มีไข้ ไอ เจ็บคอ นํามูกเล็กน้ อยมา 2 วัน ช่องคลอดและมีก้อนเลือดหลุดออกมาด้ วย ผลตรวจทางพยาธิ ซือยากินเอง วันนีมีไข้ สง ซีด ปัสสาวะสีดา ตรวจร่างกายพบ ู ํ วิทยาพบเป็ น chorionic villi ผู้ป่วยรักษาตัวอยู่ 15 วัน สามารถ moderate pale, no jaundice, no hepatosplenomegaly, pharynx NLE_step_2_2010_NCTMS Page 3
  • 4. and tonsils are injected ส่ง investigation ใดเหมาะสมทีสุดใน 25.ผู้ป่วยเด็กชายอายุ 2 ปี กินเก่งมากขึนมา 2 เดือน V/S BP การวินจฉัย ิ 140/100, HR 100, RR 24, BW 25 kg, Ht 90 cm GA - A. UA Generalized obesity, Hirsutism Heart&Lung – WNL Abd - B. reticulocyte count purplish striae จงให้ การวินจฉัยผู้ป่วยรายนี ิ C. complete blood count A. Exogenous steroid D. G6PD level B. Prader-Willi's Syndrome E. peripheral blood smear C. Neuroblastoma 21.ผู้ป่วยอายุ 5 ปี มจุดเลือดออกตามตัว ไม่มไข้ PE: Generalized ี ี D. Pheochromocytoma multiple supreficial ecchymosis, otherwhise is E. Cushing Syndrome normal ,CBC: Hct 34-37%,WBC 6000,N 60-65%,L 26.ผู้ป่วยเด็กชายอายุ 5 ปี มีอาการไข้ สงทานยาลดไข้ แล้ วไม่ลดลง ู 20-27%,E 12-15%,plt 200000,Bleeding time >15 min มา 5 วัน ไอ ปวดหัว เพลีย ปวดท้ อง มีอาการคลืนไส้ อาเจียน ให้ management 3-4 ครัง มีผนตามร่างกาย ตรวจร่างกาย วันนีไข้ ลง BT 37 ื A. ส่ง ANA BP 90/70 mmHg PR 120/min RR 32/min BW 20, look B. stool occult blood weak, flush face, liver 4 cm BRCM petechiae both arms and C. Pednisolone legs จงให้ การรักษาทีเหมาะสมในผู้ป่วยรายนี D. Platelet transfusion A. 5% DN/3 rate 60 cc/hr E. Cryoprecipitate B. 5% DN/2 rate 100 cc/hr 22.ผู้ป่วยเด็กชายอายุ 16 ปี จําเลือดเป็ นๆหายๆ ครังนีมาด้ วยมีเข่า C. 5% DN/2 rate 200 cc/hr บวม 1วัน PE: warm+swelling right knee Lab: CBC,Plt ไม่ตา ํ D. 5% DNSS rate 100 cc/hr PT ปกติ PTT 68 manage เบืองต้ น??? E. 5% DNSS rate 200 cc/hr A. DDAVP 27.ผู้ป่วยเด็กชายอายุ 4 ปี จมนํา ไม่ร้ สติ PE : mild cyanosis BT ู B. Cryo 36.5-37.5 BP 90/60 HR 140 RR 50 fine crep+wheeze C. FFP both lung O2sat (O2 10 LPM ) = 90% จง manage D. Cryo-removed plasma A. NB beta2 agonist E. Factor VII B. NB adrenaline 23.ผู้ป่วยเด็กชายอายุ 8 ปี มีผนขึนตัวลามไปแขนขา หลังทาน ื C. IV dopamine อาหาร BP 80/50ปวดท้ องอาเจียน lung: no wheezing D. O2 mask c bag Mx??? E. Intubation c ventilator support A. Adrenaline IM *** F. NSS B. CPM IV 28.ผู้ป่วยเด็กชายอายุ 5 ขวบ ถูกนําส่งมาด้ วยทีมีอาการจมนําในสระ 24.ท่านเป็ นแพทย์ประจํารพ. ผู้ปกครองมาขอปรึกษาเมือ 3 ว่ายนํา ไม่ร้ สกตัว ตรวจร่างกายพบ central and peripheral ู ึ สัปดาห์หลังจากรับทราบผลการตรวจเลือดของลูกตอนแรก cyanosis ให้ initial management คลอด 3 วัน ว่ามี TSH ผิดปกติ...ท่านจะทําอย่างไร A. CPR A. เจาะ thyroid antibody ของแม่ B. Respiration support B. เจาะ T4, TSH ลูก 29.ผู้ป่วยเด็กชาย preterm BW = 2200 gm มารดาคลอด C/S C. เจาะ thyroglobulin ของลูก due to abruption placenta apgar score 5, 8 หลังคลอด 12 ชม D. ทํา thyroid ultrasonography เหลือง ทํา exchange สาหตุสาคัญของการทีต้ องตรวจการได้ ยิน ํ E. ให้ thyroxine ของเด็กคนนี คือ A. BW = 2200 NLE_step_2_2010_NCTMS Page 4
  • 5. B. abruptio placenta D. Chest compression C. apgar score 5,8 E. Adrenaline iv D. preterm 36.เด็กเหม่อ อาการ Absance รักษาอย่างไร E. เหลือง Ans. Na valproate 30.เด็กครบกําเนิด หนัก3400กรัม เหลืองที10ชัวโมง MB 37.เด็กแรกคลอด ไม่ได้ LMP 12 Hr หลังคลอด มีหายใจหอบ 10 hct 38 เด็กเลือดกรุ๊ป เอ direct coomb weekly+ve แม่ เหนือย BW 3200 RR 60 bpm ผิวหนังเหียวย่น เล็บยาว มี เลือดกรุป โอ indirect coomb –ve ทําอย่างไร ๊ meconium stain fluid Hct70% DTX 50 ถามว่าเด็กคนนีเป็ น A. Hydration อะไร B. PRC transfusion A. Pneumonia C. exchang transfusion B. Polycythemia D. intensive phototherapy C. Hypoglycemia E. avoid breast feeding D. RDS 31.NB 6 hr ซึมลง MAP 38 mmHg ทําไร E. MAS A. Normal saline iv load 38.ผู้ป่วยเด็กชายอายุ 1 ขวบ ไอเสียงก้ อง รักษา? B. Dobutamine Ans.Epinephrine NB 32.NB หนัก 4200 gm ที 2 ชม.มีอาการกระตุก แขน ขา เจาะ BS 39.ผู้ป่วยเด็กชายอายุ 10 เดือน มีไข้ ประมาณ 38 มีหายใจขอบ ได้ 37 ให้ 10%glucose จนเพิมเป็ น 25% glucose Plasma เหนือย หายใจเร็ว ฟัง lung ได้ wheezing มาพบแพทย์ ได้ พ่น glucose 50 mg/dl น่าจะเกิดจากสาเหตุใดทีสุด ยาขยายหลอดลมไป สองครัง อาการไม่ดขน นึกถึงอะไร ีึ A. Hypopituitary A. Asthma B. hyperinsulinemia B. Pneumonia C. Hypercortisol C. Croup D. Adrenal insufficiency D. bronchiolitis E. Alocohol withdrawal syndrome 40.ผู้ป่วยเด็กชายอายุ4ปี พ่อเป็ นTB on ยา อยู่ 1 สัปดาห์กอน ลูก ่ 33.Case TOF เขียว เหนือย central cyanosis Culture→Entero ไม่มอาการอะไรเลย ตรวจร่างกายปกติ ตรวจPPD negative ถาม ี spp.ให้ ATB? management? Ans.Vancomycin A. ไม่ทาไรเลย ํ 34.ผู้ป่วยเด็กชายอายุ 1 ปี มีอาการชัก เขียว หมดสติ ตรวจร่างกาย B. INH prophylaxis 3 mo พบ flacid, cyanosis, no heart sound ควรทําสิงใดเป็ นอันดับ C. INH prophylaxis 9 mo แรก D. INH + rifampin 3 mo A. Defibrillation E. INH + rifampin 9 mo B. chest compression 41.ผู้ป่วยชายอายุ 18 yr หมดสติ หลังงานรับน้ อง ได้ กลินเหล้ า PR C. ambu bag with mask 100%O2 60 BP100/60 D. IV adrenaline A. Ketone level E. intubation B. Electrolyte 35.เด็ก 1 ปี ชักเกร็ง ส่งER ไม่ร้ สกตัว Cyanosis no heart sound ูึ C. CT brain ถาม First management ? D. 50% glucose IV A. O2 mask 100% E. Atropine IV B. ET tube 42.ผู้ป่วยชายอายุ 77 ปี หมดสติ คลํา carotid pulse ไม่ได้ ทํา chest C. Defibrillation compression ไป ekg : asystole ทําอะไรต่อ NLE_step_2_2010_NCTMS Page 5
  • 6. A. Adrenaline*** 47.ผู้ป่วยหญิงอายุ 65 yr U/D HTN ,glaucoma ซือยากินเอง ญาติ B. Amiodarone พบหมดสติ ไม่ร้ สกตัว PE: ซึม, BP 140/90 ,disorentated to ุึ C. Defrib 200 J time-place-person, Lab : hypoNa ,hypoK, hypoCl, CO2 35 D. Cardioverse 100 J BUN 40 Cr 1.6 ถามว่าอาการดังกล่าวเกิดจากยาตัวใด 43.ผู้ป่วยหญิงอายุ 18 ปี มีเลือดออกตามไรฟัน ประจําเดือนมานาน A. acetazolamide กว่าปกติ มีจุดเลือดออกทีขา ไม่ซด ตับม้ ามไม่โต CBC Hb12.2 ี B. HCTZ WBC5000 Plt50000 PTT40 PT30 ถามว่าเป็ นอะไร C. spinorolactone A. MDS 48.ผู้ช่วยพยาบาลประจําห้ องฉุกเฉินโดนเข็มตําจากการเจาะเลือด B. DIC ผู้ป่วยเมาสุรา มีเลือดออกเล็กน้ อย ได้ ทาแผลและใส่ betadine ํ C. TTP แล้ ว การปฏิบติตัวทีเหมาะสม สําหรับผู้ช่วยพยาบาลรายนีคือข้ อ ั D. ITP ใด 44.ผู้ป่วยชายอายุ 18 ปี ดืมนํามาก ปัสสาวะมาก 2เดือน ปวดหัว A. ขอผู้ป่วยตรวจเลือดหา Anti-HIV ตามัว 1เดือน MRI brain : Suprasellar mass E’lyte : Na 145 B. เริมกินยา Antiviral ได้ ทนที ั K 3.8 Cl 110 CO2 21 fluid input/output = C. รอตรวจเลือดซําอีก 6 สัปดาห์ 5100:5800manage D. ส่งเลือดผู้ป่วยตรวจหา Anti-HIV A. Restrict fluid E. แนะนําว่าโอกาสติดเชือน้ อยมาก B. Restrict Na 49.ชาย 75 ปี เก็บเหล็กขาย มีอาการนอนไม่หลับ กระวนกระวาย C. Hydrocortisone กระส่าย กระสับ ปวดท้องรุนแรงเป็ นพักๆ 3เดือน PE: marked D. Hctzz pale Ix?? E. DDAVP A. LFT 45.ผู้ป่วยชายอายุ 25 ปี ไอเรือรัง AFB 3+ ,CXR : Cavity with B. Blood lead level*** surrounding granuloma Dx Pulmonary TB ได้ IRZE 2 เดือน C. EKG F/U ได้ AFB 1+ ทําไรต่อ D. CT Abdomen A. ลดเหลือ IR 50.ผู้ป่วยชายอายุ 30 ปี เหนือย อ่อนเพลีย BP 160/110 PR 70 B. ลดเหลือ IRE อืนๆปกติ Na 154 K 2.2 HCO3 32 การวินจฉัยทีเหมาะสมคือ ิ C. ให้ IRZE ต่อ 1 เดือน A. Essential HT D. Add streptomycin B. Hyperthyroid E. Add ofloxacin C. Pheochromocytoma 46.ผู้ป่วยหญิงอายุ 25 ปี ไข้ ไอ เหนือย 5 วัน ไอเสมหะเหลือง ไป D. 1st hyperaldosterone ซือ Amoxycillin500 1×3 กินเอง 3 วัน ไม่ดีขน PE: ึ E. Cushing syndrome BT36.5°C,PR 102,RR 26,BP135/70Lung:Dullness on 51.ผู้ป่วยหญิงอายุ 68 yr U/D HT กินยาไม่สมําเสมอ มาด้ วยหอบ percussion right lower lung and decrease breath sound and เหนือย PE: BP200/110 PR 100 neck vein engorge S3 vocal resonance right lower lung รักษาอย่างไร gallop fine crep both lung Ext: pitting edema lower leg Eye: A. เพิม Amoxycillin exudates c wire ring Mx?? B. เพิม Clarithromycin A. Digoxin C. เปลียนเป็ น Levofloxacin B. Propanolol D. Thoracentesis C. Nifedipine E. ICD D. Nitroglycerine NLE_step_2_2010_NCTMS Page 6
  • 7. 52.DM ได้ MFM 1500,Glipizide 15 BMI เยอะ FBS 180 Ans.Indommmethacin HbA1C 8.1 62. HIV เป็ น Meningitis CSF glucose ลด Protein เพิม CD4>250 A. เพิม Glipizide Indian ink neg การวินจฉัยทีเหมาะสมคือ ิ B. MFM 2000 Ans.TB meningitis C. add acarbose 63. ผู้ชายปวดหัว ปวดเบ้ าตาจนต้ องตืนกลางดึก มีอาการนาน 2 D. ให้ RI เดือน แล้ วหายไป ตอนนีกลับมาเป็ นใหม่มีปวดเบ้ าตาซ้ ายบ้ าง 53.ผู้ป่วยชายอายุ 70 ปี CC: ผล Lab ผิดปกติ ทานวิตามินเสริม หรือเบ้าตาขวาอย่างเดียว การวินจฉัยทีเหมาะสมคือ ิ เป็ นประจํา 10 ปี ก่อนเคยปัสสาวะปนทราย PTH 7, Ca2+11, A. Cluster headache PO4 2- 3.5, Cr 1.1, การวินจฉัยทีเหมาะสมคือ ิ B. Tension headache A. HyperCa2+ of Malignancgy C. Migraine B. CA with bone Met. 64.SLE เป็ น Pregressive GN on cyclophosphamide แล้ วมีไข้ ตุ่ม C. 1OHyperparathyroid แดงต่อมามีสดาตรงกลางคล้ าย Eschar รักษา? ี ํ D. 2OHyperparathyroid Ans.Doxycyclin E. 3OHyperparathyroid 65. DM HT DLP กินยาบํารุงหลายอย่าง PTH สูง Ca สูง 54.EKG bradycardia Mx A. Vit D intox Ans:Atropine B. 1° hyperparathyroid 55.กินหน่อไม้ ดอง + เหล้ า เกิด Respiratory… C. 2° hyperparathyroid Ans : Botulinum toxin D. 3° hyperparathyroid 56.EKG Asystole Mx 66. รูปงู ผล Lab ปกติมี Local wound swelling,Hemorrhagic bleb Ans:Adrenaline ทําไร 57.Hypo K, Hypo Na A. Antivenum Ans : 1o Hyperaldosteronism B. TT 58.ผู้ป่วยหญิงอายุ 80 ปี มาด้ วยอ่อนแรงซีกซ้ าย หมดสติ 5 day C. Broad spectrum ATB PTA ขณะนังดูโทรทัศน์ PE: stupor HR 60 totally irregular 67.ผู้ป่วยชายอายุ 70 ปี ไม่มโรคประจําตัว lipid profile TG 500 ี RR 12 Lt.hemiparesis pupil 2 mm Lt. 4 mm Rt. Upgoing Choles 210 HDL 40 Lab อะไรผิดปกติด้วย Lt.plantar reflex ท่านเป็ นแพทย์ที ER จง Manage เบืองต้ น A. Uric acid อย่างไร B. FBS A. IV furosemide C. Cortisol B. IV nitroglycerine D. Thyroid function test C. IV heparin drip 68. ผู้ป่วยหญิงอายุ 16 ปี ล่องเรือไปเทียวเกาะ ขณะเรือออกกลาง D. Et tube ทะเลมีคลืนไส้ อาเจียน ใช้ ยาอะไร E. IV Dexamethasone A. Paracetamol 59.ผู้หญิงมีรอยนําตาลนูนมา 5 วัน ตรงกลางมีผมร่วงในแผลเห็น B. Dimenhydrinate เส้ น เลือดชัดเจน (Telangiectasia) C. Propanolol Ans.Discoid LE D. tramadol 60.ผู้ป่วยหญิงอายุ 65 ปี DM HT on ยา HCTZ ACEI MFM แล้ ว E. Cinnarizine กิน Allopurinol Para แล้ วมีผน MP rash เกิดจาก? ื 69.ผู้ป่วยหญิงอายุ 75 ปี U/D HT on ยา HCTZ (25) 1x1 Ans.Allopurinol ควบคุมความดันได้ ปกติ ตอนเช้ าลุกจากทีนอนไม่ขน ญาตินาส่ง ึ ํ 61.อาการ Gout เป็ นครังแรก รักษา? รพ. PE: Alert, BP 130/80 mmHg, PR 72, RR 18, BT 37 NLE_step_2_2010_NCTMS Page 7
  • 8. Proximal m. weakness, normal sensation and reflex อาการ C. ASA + CCB ของผู้ป่วยน่าจะเกิดจากอะไร D. ASA + BZD A. HypoNa+ E. beta-block + heparin B. HypoK+ 74. ผู้ป่วยหญิง ซีด อ่อนเพลีย CBC เป็ น iron def ได้ รับการรักษา C. HypoMg2+ ด้ วยยาเม็ดสีนาตาลดํา ถามว่าหลังได้ รับยาจะติดตามการรักษา ํ D. HyperCa2+ ด้ วยวิธใดได้ เร็วทีสุด ี E. Rhabdomyolysis A. Hb 70. ผู้ป่วยชายอายุ 65 ปี เป็ นมะเร็งปอดระยะที 1 ได้ รับการผ่าตัด B. MCV ไปแล้ ว หลังผ่าไม่ได้ รับเคมีบาบัดหรือรังสีรักษา ํ C. Recticulocyte count ต่อมามีอาการซีกขวาอ่อนแรง gr.4 ร่วมกับมีอาการชักเกร็งทัว D. Serum ferritin ตัว ก่อนมา 30 นาที ชักเกร็งกระตุกจากแขนขา แล้ วต่อมาชักทัว E. Serum haptoglobin ตัว หลังชักรู้สกตัวดี PE : good conciousness , Lung - clear , ึ 75.ผู้ป่วยชายไทยมีไข้ และเจ็บคอมา 1 wk ซือยามากินเองอาการไข้ scar at Lt. chest wall , tracheotomy scar จะให้ ยาอะไร ดีขนแต่มอาการอ่อนเพลียมากขีนมา 3 d PE: v/s ปกติ , pale ึ ี A. Clonazepam conjunctiva, icteric sclera , spleen 2 FB BLCM LAB: Hct B. Phenobarbital 25% MCV 65 plt 400000 WBC 9600 N 60% L 40% C. Phenytoin ถามว่า diagnosis อะไร D. Carbamazepine A. Hb H disrase 71. ผู้ป่วยชายอายุ 75 ปี no underlying. ไม่ได้ รับประทานยาใด B. G6PD รับประทานอาหารได้ ไม่มนาหนักลด ท้องผูกเป็ นประจํา ตรวจ ี ํ C. HS ร่างกายพบ V/S BP 140/80 PR 84 RR 18 (mild)pale, D. AIHA total dental loss, glossitis, koilonychiae จงให้ การวินิจฉัยที E. PNH เป็ นไปได้ มากทีสุด 76.ผู้ป่วยหญิงอายุ 30 ปี มาด้ วยอ่อนเพลีย ซีด 3 สัปดาห์ ตรวจ A. hypothyroid ร่างกายพบ moderate pale, mild icteric sclera, other within B. normal aging change normal CBC : Hct 25%, WBC 5000 cell/mm3 N68 L32, C. iron deficiency anemia plt. 200000 cell/mm3, RBC morphology : normochromic, D. anemia of chronic disease anisocytosis 2+, microspherocyte 2+, polychromasia 1+ การ E. autoimmune hemolytic anemia ตรวจใดช่วยยืนยันการวินจฉัยิ 72. ให้ ผล csf : open pressure 30 cmh20 protein 550 sugar 20 A. Hb typing ผู้ป่วยมาด้ วยอาการไข้ ปวดศีรษะ การวินจฉัยทีเหมาะสมคือ ิ B. Coomb’s test A. toxoplasmosis meningitis C. G6PD B. CMV meningitis D. Osmotic fragility C. tubeculous meningitis 77.ผู้ป่วยหญิงอายุ 70 ปี มีอาการ อ่อนเพลีย เหนือย มา 4-5 73. ผู้ป่วยชายอายุ 37 yr เจ็บกลางอกร้ าวไปไหล่ซ้าย เหงือแตก ใจ เดือน ตรวจร่างกายพบ glossitis, moderate pale, CBC : Hct สัน มีประวัติเป็ นโรคหืด ใช้ ยาขยายหลอดลมนานๆครัง ไม่มี 22%, WBC 3400, N 65%, L 30%, Plt 80000, ให้ รป ู บุคคลในครอบครัวเป็ นโรคหัวใจ PE: BP 170/85 PR 110 blood smear มา เหมือนมี hypersegmented neutrophil การ วินจฉัยทีเหมาะสมคือ ิ EKG ST elevation V2-6 การดูแลขันต้ นในผป รายนี? A. Aplastic anemia A. ASA + beta-block B. Megaloblastic anemia B. ASA + nitrate C. Iron deficiency anemia NLE_step_2_2010_NCTMS Page 8
  • 9. D. Myelodysplasia E. PE 78.ผู้ป่วยชาย อ่อนแรงมากขึนมา 3 วัน ตรวจร่างกายพบ total 82.ให้ รป blood smear มา ร่วมกับให้ อาการของ malaria มา blood ู opthalmoplegia, bilateral facial palsy, paradoxical breathing, smear มีลกษณะของ double chromatin and multiple ring บอก ั generalized muscle weakness grade II/V, areflexia, loss of ถึง P.f. ถามว่าจะรักษาด้ วยยาอะไร proprioception of fingers and toes. What is the most likely ตอบ artesunate + mefloquine diagnosis 83.หญิงอายุ 55 ปี มี U/D เป็ นโรคไตเรือรัง ฟอกไต 2 ครัง/ A. Multiple sclerosis สัปดาห์ มาด้ วยหอบเหนือย Electrolyte : Na 138, K 6.8, B. Myasthenia Gravis BUN 60, Cr 7...รักษาอย่างไร C. Guellain Barre Syndrome A. NaHCO3 D. Acute myelopathy B. Insulin + นําตาล E. Botulism C. Dialysis D. Beradual 79.ผู้ป่วยชายไทยอายุ28ปี นําหนัก 80กิโล ประสบอุบติเหตุทาง ั E. Kayexalate รถจักรยานยนต์ ได้ รับอุบตเหตุทสมอง นอนพักรักษาตัวที ั ิ ี 84.ผู้ป่วยชายอายุ 75 ปี มีอาการมือสันเวลาถือแก้ วนํา เป็ นมากขึน โรงพยาบาลมาเป็ นเวลา 7วัน ได้ รับอาหารผ่านทางNG เวลาโมโหหรือดืมกาแฟ PE: postural tremor , normal muscle tube มาตลอด โดยได้ พลังงาน 1800 kcal/day และ protein tone, no dysmentia, normal gait นึกถึงภาวะใดมากทีสุด 90 g/day ต่อมาพบว่าalbuminลดลงจาก 4.2เหลือ 2.4 ถาม A. normal change ว่าควรปรับเปลียนอาหารอย่างไร B. Parkinson A. เพิมพลังงานเป็ น 2000 เพิมโปรตีนเป็ น 120 C. cerebellar dystrophy B. เพิมพลังงานเป็ น 2400 โปรตีนเป็ น 120 D. autoimmune dysfunction C. เพิมพลังงานเป็ น 2800 โปรตีนเป็ น 140 85.ผู้ป่วยหญิงอายุ 23 ปี อาชีพครูโรงเรียนเด็กอนุบาล เคยป่ วยเป็ น D. เพิมไข่ขาว 2 ฟองต่อมือ Rheumatic fever ตอนอายุ 15 ปั ปัจจุบนทํางานทัวไปได้ ปกติ ั E. เพิมไข่ขาวและไข่แดงมือละฟอง ไม่มอาการหอบเหนือย ตรวจร่างกายพบ pansystolic murmur ี 80.ผู้ป่วยหญิงอายุ 40 ปี จะไปเทียวภูเขาหิมาลัย มาพบแพทย์เพือ grade II/VI at apex ได้ รับการรักษาด้ วย Penicillin V มาตลอด ขอคําปรึกษาและยา เพือลดอาการคลืนไส้ ขณะไปเทียว ข้ อใด ควรให้ การรักษาผู้ป่วยรายนีต่อไปอย่างไร ควรปฏิบติมากทีสุด ั A. หยุดยา penicillin A. omeprazole B. ให้ กนยาเฉพาะตอนทําฟั น ิ B. Altitude acclimation C. ให้ กนยาไปจนกระทังอายุ 30 ปี จึงหยุด ิ C. Chloroquine D. ให้ กนยาไปจนกระทังอายุ 50 ปี จงหยุด ิ ึ D. Hydrochlorothiazide E. ให้ กนยาไปตลอดชีวต ิ ิ E. Metformin 86.หญิง มาหาหมอ ปรึกษาเรืองไปเทียวหิมาลัย แล้ วไม่อยาก 81.ผู้ป่วยหญิงอายุ 20 ปี มีไข้ มาสามวัน ต่อมามีเจ็บแน่นหน้ าอก คลืนไส้ อาเจียน เหนือยง่ายขึน นอนราบไม่ได้ ตรวจร่างกาย V/S : BT 39, BP A. Melatonin 90/70,PR110 ,RR22 neck vein engorge, decrease heart B. Thiazide sound w/ scattering, lung clear ,other WNL C. Omeprazole A. CHF D. Chroroquine B. Severs MS E. Attitude acclimator C. Pericarditis 87.HBsAg + HBcIg M + Anti HBe - LFT normal Mx??? D. Myocarditis A. F/U 6 mo NLE_step_2_2010_NCTMS Page 9
  • 10. B. Start lamivudine 92.ผู้ป่วยชายอายุ 70 ปี เป็ น pneumonia มา 5 วัน มี U/D COPD C. Viral load ให้ O2 5 LPM Cef-3+azithromicin อยู่ๆก็เหนือยมาก D. Reassure ขึน และซึมลง ควรทําอย่างไร 88.ผู้ป่วยชายอายุ 45 ปี กินเหล้ า มีไข้ และเจ็บอก 1 wk ตรวจ A. intubation ร่างกายพบ pleural effusion จึงเจาะปอดได้ นาสีเหลืองขาวขุ่น ํ 93.ผู้ป่วยชายอายุ 60 ปี มาอยู่ ICU Dx pneumococcal ย้ อมแกรมได้ Gram negative bacilli, Gram positive cocci in pneumonia มา 10 วัน อาการดีขน เมือวานสําลักอาหาร วันนี ึ chain , positive bacilli จะให้ ยาตัวใด มีไข้ สง มีเสมหะและไอ BT 39.5 ,BP 90/60, PR 100, RR ู A. penicillin 30 Imaging : crepitaion at Lt. lower lung, Sputum G/S : B. ceftriaxone Gram neg. bipolar stain ให้ ATB อะไร? C. cotrimoxazole A. Amikacin D. clarithomycin B. Amoxy+Clav. E. amoxyclav C. Ciprofloxacin F. Levofloxacin D. Imipenem 89.ผู้ป่วยชายอายุ 32 ปี หอบเหนือยมา 3 วัน เป็ นหลังไข้ หวัด เป็ น E. Ceftriaxone โรคหืดมา 20 ปี ปัจจุบันได้ inhale budesonide PE: tachypnea, 94.ผู้ป่วยชายอายุ 58 ปี มีไข้ สง 3 วัน ต่อมามีหอบและซึมลง 1 วัน ู decreased breath sounds at Rt. Upper lung with dullness on มีจุดจําเลือดตามตัว Lab: Hb9.8, Hct 30% WBC 28000- percussion, rhonchi both lungs, O2 saturation = 85% การ 38000 N 83% band 10% L7% plt 15000, PT 16 PTT 60 ตรวจทางปฎิบติการใด ช่วยวินจฉัยโรคของผู้ป่วยรายนีได้ ดทสุด ั ิ ี ี fibrinogen 105 นึกถึงภาวะใดมากทีสุด A. Peak expiratory flow rate A. ITP B. Portable chest x-ray B. TTP C. Spiral CT angiography C. acquired factor VIII deficiency D. Arterial blood gas D. DIC E. Sputum Gram stain E. vWD 90.ผู้ป่วยชายอายุ 50 ปี no U/D มีประวัติสบบุหรีและกินเหล้ าเป็ น ู 95.ผู้ป่วยชายอายุ 38 ปี มีไข้ สงมา 3 d PTA ต่อมาซีมลง ตรวจ ู ครังคราว เลิกมาแล้ วประมาณ 1 ปี เพราะว่ามีปัญหา ED ตรวจ ร่างกายพบมี mild pale, no Jaundice, no hepatosplenomegaly, เลือด Chol. 210, TG 150, HDL 55, LDL 120 จะวาง Petechiae & Purpura all extremities ผล CBC พบมี Hct 28, แผนการรักษาอย่างไรต่อไป WBC 28,000, N 95%, L 5%, Band 10%, Plt 40,000 A. ไม่ทาอะไร --> normal ํ ตรวจ PBS พบมี Schistocytosis, ตรวจ CoaguloGram พบมี B. ให้ จด morning erection PT, aPTT Prolong, Fibrinogen ตํา ,ในผู้ป่วยรายนีน่าจะคิดถึง C. ให้ ยา treat ED โรคใดมากทีสุด D. consult psychiatrist A. Vit K def E. consult urologist B. primary fibrinolysis 91.ในการป้ องกันการแพร่ระบาดของวัณโรคปอดในบุคลากรทาง C. DIC สาธารณสุขทีโรงพยาบาล ข้ อใดมีประสิทธิภาพทีสุด D. TTP A. ให้ ผ้ ป่วย ทีสงสัยวัณโรคสวมหน้ ากาก ู E. Thalassemia with sepsis B. ให้ บคลากรทางการแพทย์สวมหน้ ากากป้ องกัน ุ 96.ผู้ป่วยหญิงอายุ 18 ปี เลือดออกตามไรฟัน ประจําเดือนมามาก C. ติดเครืองปรับอากาศในหอผู้ป่วย ผิดปกติ ไม่มไข้ ไม่มโรคประจําตัว จุดเลือดออกตามตัว แขน ขา ี ี D. ย้ ายเตียงผู้ป่วยวัณโรคไปอยู่ท้ายวอร์ด ตับม้ ามไม่โต Lab: Hb,Hct,WBC normal มี Eo 5% plt 15000 APTT 35,PT 12 การวินจฉัยทีเหมาะสมคือ ิ NLE_step_2_2010_NCTMS Page 10
  • 11. A. Myelo…. 100. ผู้ป่วยชายอายุ 18 ปี ถูกทําร้ ายร่างกาย หายใจเหนือย ไอเป็ น B. TTP เลือด เสียงแหบ มี subcutaneous emphysema at neck , air C. ITP hunger management คือ D. APDE A. Intubation E. DIC B. CT larynx 97.ผู้ป่วยหญิงไทย มาโรงพยาบาลด้ วยเรืองกินนํายาล้ างห้ องนํา C. Tracheotomy โจทย์บอกว่าเป็ นด่าง v/s ปกติ (ไม่แน่ใจนะ >>มีแสบบริเวณ D. barium swallowing หน้ าอก และลําคอ) คําถามคือ ควรทําอย่างไร E. ABG A. กระตุ้นให้ อ้วก 101. ผู้ป่วยหญิงอายุ 30 ปี ล้ มหน้ าอกขวากระแทกโต๊ะ 30 นาที B. ให้ charcoal ต่อมามาโรงพยาบาล PE : PR 80 RR 30 BP 110/80 CXR C. ให้ NaHco3 : right 4th&5th rib fracture with pneumothorax 15% หลัง D. ใส่ NG ได้ O2 แล้ วทําอะไรต่อ E. Early scope A. closed observe 98.ผู้ป่วยชายอายุ 26 ปี โดนสุนขกัดเป็ นรอยถลอก ไม่มเลือดออก ั ี B. pain control หลังจากทําความสะอาดแผลแล้ ว ควรทําอย่างไร C. needle thoracotomy A. สังเกตพฤติกรรมสัตว์อย่างเดียว D. thoracotomy B. สังเกตพฤติกรรมสัตว์ และให้ ยาปฏิชวนะ ี E. ICD C. ให้ วคซีนพิษสุนขบ้ าจนครบ course ั ั 102. ผู้ป่วยชายอายุ 26 ปี MCA หน้ าอกกระแทก V/S ………….. D. ให้ วคซีนพิษสุนขบ้ า และให้ ยาปฏิชีวนะ ั ั PE: (บรรยายมาเข้ าได้ กบ Beck ‘s triad ) ให้ o2 canula และ ั E. ให้ วคซีนพิษสุนขบ้ าและ RIG ั ั IV saline ไปไม่ดีขน จะทําอย่างไรต่อ??? ึ F. ให้ tetanus toxoid จนครบ course A. ให้ inotrope drug G. ให้ tetanus toxoid + TAT จนครบ course B. pericardiocentensis H. ให้ ยาปฏิชีวนะอย่างเดียว 103. ผู้ป่วยชายอายุ 65 ปี ถ่ายปนเลือดเป็ นๆหายๆมา2สัปดาห์ วัน I. ให้ tetanus toxoid + TAT จนครบ course ร่วมกับยา ละ2ครัง ลักษณะเป็ นอุจจาระปนเลือดสีดาคลําปนมูก ตรวจ ํ ปฏิชีวนะ abdomenปกติ rectal examinationปกติ proctoscopeพบ 99. ผู้ป่วยชายอายุ 40 ปี ถูกมีดบาดขณะปอกผลไม้ 1 ชัวโมงก่อน อุจจาระสีเหลืองปนมูก ไม่มแผล ไม่มก้อน ไม่พบความ ี ี มาโรงพยาบาล แผลกว้ าง 1 cm ลึก 0.5 cm มีเลือดออก ผิดปกติอนๆ ควรทําอะไรต่อไป ื เล็กน้อย ได้ ทาการล้ างแผลด้ วยสบู่ และทาเบตาดีนเองจาก ํ บ้ าน ผู้ป่วยกลัวว่าจะเป็ นบาดทะยัก จึงมาขอฉีดยากัน A. bulk forming laxative บาดทะยัก ; 4 ปี ก่อนมาโรงพยาบาล ผู้ป่วยเคยถูกตะปูตาทีเท้ า ํ B. fecal occult blood ได้ รับการฉีดยากันบาดทะยัก 3 เข็ม ทีโรงพยาบาลทีท่าน C. barium enema ปฏิบติงานอยู่ การปฏิบติทเหมาะสมทีสุด สําหรับผู้ป่วยราย ั ั ี D. CEA นีคือข้ อใด 104. colonoscopyผู้ป่วยชายอายุ 50 ปี มีไข้ ปวดบันเอวขวาร้ าวไป A. ไม่จาเป็ นต้ องฉีดยากันบาดทะยัก ํ หลังและขาหนีบ ปัสสาวะแสบขัด เป็ นๆหายๆ 2 ครัง ไม่เคย B. ฉีด Tetanus toxoid 1 เข็ม ถ่ายปัสสาวะเป็ นเลือด ตรวจร่างกายปกติUA : WBC 0- C. ฉีด Tetanus toxoid 3 เข็ม 1 RBC 10-20 Investigation ทีช่วยในการวินจฉัย ิ D. ฉีด Tetanus antitoxin ร่วมกับ Tetanus toxoid 1 เข็ม A. urine calcium E. ฉีด Tetanus antitoxin ร่วมกับ Tetanus toxoid 3 เข็ม B. IVP C. cystoscope NLE_step_2_2010_NCTMS Page 11
  • 12. D. u/s KUB D. U/S Abdomen E. plain KUB E. BE 105. ผู้ป่วยชายมีประวัติสบบุหรี ปัสสาวะเป็ นเลือดสีแดงตลอดสาย ู 110. ผู้ป่วยชายอายุ 70 ปี มาด้ วยปวดท้องทันที 2 ชม. PTA ไม่ปวดท้ องน้ อย ไม่ต้องเบ่งเวลา ถ่ายปัสสาวะ investigation? BP170/80, PR120 bpm irregular pulse PE: Abd soft, not A. CT whole abdomen tender, absent bowel sound CBC ไม่ซีด WBC21,000 B. Retrograde urethroGram Neutrophil เด่น การวินจฉัยทีเหมาะสมคือ ิ C. Cystoscopy A. PU perforate D. VCUG B. Acute pancrsatitis E. IVP + Cyrtouroscope C. Rupture appendicitis F. CystouretheroGram D. Sup. mesenteric a. G. Voiding cystouteroGram 111. ผู้ป่วยหญิงอายุ 18 ปี MCA PE: Closeก Fx of femur BP H. CysouretheroGram +IVP 80/50 PR 120 GCS 14 มี Abrasion at right chest 106. ผู้ป่วยชายถ่ายเป็ นเลือดคลํา มานาน PE: ปกติ Abdomen : tender at right abdomen หลัง Load NSS แล้ ว Procto+Sigmoidoปกติ, -ve bowel habit change ทําอะไรต่อ V/S ยังไม่ดขน ทําไร ีึ A. Colonoscopy A. Observe B. AngioGram B. FAST C. Abd tapping C. DPL 107. ไม่มอาการ เจาะ PSA ได้ 3 PR- prostate ปกติ ทําอะไร ี D. CT A. TURP E. Explor.lap B. Transrectal U/S 112. ผู้หญิงถ่ายอุจจาระเป็ นเลือด บางครังปนเลือดสด PR ปกติ Ix C. PR q 1 yr + PSA q 1 yr อะไร 108. ผู้ป่วยชายอายุ 70 yr มีปัสสาวะลําบาก กระปริบกะปรอย A. Colonoscopy ติดขัด ใช้ เวลานาน ไม่พ่งมานานประมาณ 2-3 เดือน, 3 ุ B. Sigmoidoscopy DPTA ผู้ป่วยเป็ นหวัดคัดจมูก กินยาลดนํามูก, 1 DPTA ผู้ ู C. ปลาปักเป้ า →Terotoxin ป่ วยมาด้ วยปัสสาวะไม่ออก PE : full bladder , PR: 113. ผู้ป่วยชายอายุ 20 ปี ถูกพบหมดสติในอาคารไฟไหม้ PE: PR prostate gland enlargement ,rubbery, smooth surface 64 BP 90/70 BP=90/60mmHg RR=12 /min unresponse normal anal sphincter tone Mx?? to deep pain, black soot in mouth and throat heart and A. คาสายสวน 3-5 วัน lungs unremarkable, no skin burn ส่งตรวจทาง B. คาสายสวน 3-5 วัน ให้ ยารักษาต่อมลูกหมากโต ห้ องปฏิบติการณ์ใดทีช่วยในการวินจฉัยโรคมากทีสุด ั ิ C. คาสายสวน 3-5 วัน และเตรียมผ่าตัดต่อมลูกหมาก A. pulse oximetry D. Cystostomy B. serum calcium E. ควรส่องกล้ อง cystoscopy เพือหาสาเหตุของอาการ C. serum electrolyte ปัสสาวะไม่ออก D. serum lactate 109. ผู้ป่วยชายอายุ 50 ปี ปวดลินปี 24 hr ก่อนมา รพ. จากนัน E. venous blood gas ย้ ายมาปวดทีท้ องน้ อย ด้ านขวากดเจ็บ CBC พบ 114. ผู้ป่วยหญิงอายุ 50 ปี เป็ นโรคเบาหวานมาหลายปี มีอาการไข้ Leukocytosis ส่งตรวจอะไรเพิมเติม? สูงหนาวสัน ปวดท้ อง มา2วัน PE:tenderness RUQ ให้ ภาพ A. Acute abdomen series film abdomen seriesมา(ซึงเราดูไม่ออกง่ะว่าเป็ นอะไร) ถามว่า B. CT abdomen เป็ นโรคอะไร C. GI follow through A. gastric perforation NLE_step_2_2010_NCTMS Page 12
  • 13. B. ruptured duodenum 119. ผู้ป่วยหญิงอายุ 35 ปี ทํางานพิมพ์คอมพิวเตอร์ มีอาการชา C. gut obstruction บริเวณนิวโป้ งชี กลาง ตรวจร่างกายไม่มี hypothenar wasting D. emphysematous cholecystitis ไม่มอาการปวดร้ าวไปคอ จงให้ การวินจฉัยผู้ป่วยรายนี ี ิ E. liver abscess A. Carpal Tunnel Syndrome 115. ผู้ป่วยหญิงอายุ 45 ปี มีอาการปวดท้ องด้ านบนขวาอย่าง B. DeQuarvain Syndrome เฉียบพลันมา 1 ชัวโมง สักครู่เดียวอาการก็หายไป ตรวจร่าย C. Polyneuropathy กายปกติดี ตรวจท้ อง : no mass กดเจ็บที RUQ Lab CBC D. C-Spine Sublaxation ปกติ , UA ปกติ ถามว่าจะสงตรวจอะไรเพิมเติม 120. Common Peroneal injury หลัง Stimulation ทําอะไร A. U/S A. Isotonic Exercise B. Plain KUB B. Isometric Ex. C. Plain Abdomen C. Active range of motion D. CT abdomen D. Active… E. MRI E. Passive… 116. ผู้ป่วยหญิงอายุ 55 ปี ได้ รับอุบติเหตุทสะโพกหลังการรักษา ั ี 121. ผู้ป่วยชายอายุ 60 ปี ได้ รับการวินจฉัยเป็ นเบาหวานครังแรก ิ ด้ วยวิธดึงข้ อ อาการปกติดี ต่อมามีอาการปวดทีสะโพกและ ี สูบบุหรี 2 ซอง/วัน มานาน 40 ปี เมือให้ คาแนะนําให้ ผ้ป่วย ํ ู เดินลําบาก ข้ อใดจะช่วยในการวินจฉัยและวางแผนการรักษา ิ เลิกบุหรี ผู้ป่วยบอกว่ายังไม่พร้ อม ข้ อใดควรปฏิบตมากทีสุด ั ิ มากทีสุด A. ให้ กาลังใจว่าผู้ป่วยสามารถทําได้ ํ A. Arthocentesis B. บอกข้ อเสียของการสูบบุหรี B. Film boht hip C. แนะนําวิธการเลิกบุหรีทีเหมาะสม ี C. CT D. MRI D. บอกข้ อดีข้อเสียของการเลิกบุหรีแต่ละวิธี E. Bone scan E. บอกให้ มารับคําแนะนําได้ เสมอเมือต้ องการ 117. ผู้ป่วยชายอายุ 3 ปี แม่พามาตรวจด้ วยเรืองก้ อนในท้ องข้ าง 122. ผู้ป่วยชายอายุ 45 ปี เป็ น AIDs มา 5 ปี 2 wkก่อนมีไข้ ซ้ าย ถามว่าจะ investigate อะไร หอบ พบว่าเป็ น PCP รักษามา 1wk อาการหอบดีขน ึ A. film abdomen ตอนีมีสบสน disoriented to time-place-person นอนไม่หลับ ั B. u/s กระสับกระส่าย ให้ วนจฉัย ิ ิ C. CT A. acute psychosis D. MRI B. delirium 118. ผู้ป่วยชายอายุ 70 ปี no underlying. ไข้ หนาวสัน 2 วัน ปวด C. AID dementia ท้ องบริเวณใต้ ชายโครงขวา PE: BT 38 c mild icteric sclera, D. Pseudodemantia not pale, mild abdominal distendion, liver just palpable, E. schizophrenia tenderness on RUQ percussion จงให้ การวินจฉัยทีเป็ นไป ิ 123. ผู้ป่วยชายอายุ 75 ปี เป็ นทหารอยู่ภาคใต้ ปราบปราม ได้ มากทีสุด ผู้กอการร้ าย โดนยิง เข้ ารับการรักษาด้ วยการผ่าตัด หลังผ่าตัด ่ A. liver abscess แผลหายเป็ นปกติ แต่มอาการกลัวเสียงดัง นอนไม่หลับ ไม่ ี B. cholecystitis สนใจสิงแวดล้ อมรอบตัว ข้ อใดคือ diagnosis C. cholangitis A. Acute stress response D. gallstone pancreatitis B. Post traumatic Stress Disorder E. cholangiocarcinoma C. Normal reaction D. Brief Psychotic Disorder NLE_step_2_2010_NCTMS Page 13
  • 14. E. Adjustment disorser 128. ผู้ป่วยชายอายุ 79 yr หลงลืมมา5ปี จําลูกไม่ได้ โวยวาย ไม่ 124. ผู้ป่วยชายอายุ 45 ปี ดืมเหล้ าขาววันละ 1 ขวด มา 25 ปี ถ่าย นอน เห็นภาพหลอนเป็ น สามีทตาย PE: disoriented ี เหลวมา 3-4 ครัง/d เป็ นเวลา 2 วัน มาด้ วยอาการซึมลง impaired short&long memorycalculate&language deficit สับสน จําญาติไม่ได้ มา 3 วัน ระหว่างนีกินเหล้ าตลอด ตรวจ Tx?? ร่างกาย V/S ปกติ PE: stupor, Erythematous velvety A. Vit.E hyperpigmented Desquamous scaling plaque at face neck B. Gingko and both forearms การวินจฉัยทีเหมาะสมคือ ิ C. Donepezil A. Delirium tremens D. Diazepam B. Subdural hematoma E. Haloperidol C. Pellagra 129. ผู้ป่วยชายอายุ 14 ปี บิดามาพบแพทย์ เคยโดนครูเรียกบ่อยๆ D. Electrolyte imbalance ลูกโดดเรียน สูบบุหรี วิวาท เด็กไม่ทราบสาเหตุทมาพบแพทย์ ี E. Wernicke's encephalopathy Mx??? F. Alcohol intoxication A. ถามความต้ องการทีแท้ จริงของเด็ก G. Hepatic encephalopathy B. ให้ บดาบอกเด็กตามจริง ิ 125. ผู้ป่วยหญิงอายุ 79 ปี แข็งแรงดี หลงลืมมา 8 เดือน จําไม่ได้ C. สร้ างความสัมพันธ์กบเด็ก ั เก็บเงินไว้ ทไหน ทําอาหารลืมปิ ดเตา เคยคิดว่าอยู่บ้านคนอืน ี D. ปิ ดเป็ นความลับ ทังทีอยู่บ้านของตน มีหงุดหงิด นอนหลับๆตืนๆ PE: normal E. ส่งปัสสาวะตรวจ Mental status: impair orentiated to time, impair short and 130. ผู้ป่วยชาย smoke 20 มวน/วัน U/D DM แพทย์แนะนําหยุด long term memory, impair calculate and language ถาม บุหรี แต่ผ้ ูป่วยไม่พร้ อม แพทย์ควร diagnosis ? A. บอกข้ อเสียบุหรี A. normal age change B. บอกวิธการเลิกี B. mild cognitive dysfunction C. สามารถกลับมาปรึกษาใหม่ได้ C. dementia D. บอกข้ อดีข้อเสียของบุหรีแต่ละวิธี D. derilium 131. ผู้ป่วยหญิงอายุ 22ปี มีประวัติซึมเศร้ ามา 1ปี 3สัปดาห์กอน ่ E. depression คุยเก่ง ซือของมาก คุยกับเพือนว่ามีแฟนเป็ นนักร้ องดัง โทรจิต 126. ผู้ป่วยชายอายุ 35 ปี ผ่าตัดไส้ ติง หลังผ่าตัด 3 วัน เห็นสาย คุยกันทุกวัน ให้ ยา อะไร นําเกลือเป็ นงู มีประวัติกนเหล้ ามา 10 ปี จงวินจฉัย... ิ ิ A. Lithium + Valpoic A. Delirium tremens B. Lithium + Fluroxetine B. Delusion disorder C. Valpoic + Hadol C. Schizophrenia D. Hadol + Amitryptiline D. Hepatic encephalopathy E. Hadol +… E. Hypochondriasis 132. ผู้ป่วยชาย มาด้ วยชัก ไม่เคยชักมาก่อน กินเหล้ า มา 20 ปี 127. ผู้ป่วยชายอายุ 65 ปี กินเหล้ าทุกวัน มาadmit กระดูกข้ อเท้ า สามวันที ผ่านมากินเหล้ าทุกวัน ชักมา 15 นาทีแล้ วมารพ. หัก 3 วัน เพ้ อคลัง ปี น เตียง คิดว่าจะมีคนทําร้ าย จะให้ อะไร มาถึงก็ยังชักอยู่ Mx?? A. midazolam A. Diazepam IV B. diazepam B. ETtube C. Haloperidol C. Phenytoin D. Clonazepam E. Chlorpomazine NLE_step_2_2010_NCTMS Page 14